Вы находитесь на странице: 1из 52

on his back.

He gave history of chicken pox infection at ten


years of age. The likely diagnosis was:
COMMUNITY MEDICINE A. Measles
B. Meningococcemia
C. Scarlet fever
D. Herpes zoster

1. A woman brought her child with congenital anomalies of KEY: D


heart and cataract. She gave history of mild fever and rash
in the first trimester of pregnancy, which settled with mild 8. A patient who has come from India, reports to a health
antipyretics. The possible gestational condition that resulted facility with severe generalized aches and pain and rash on
in these anomalies was: the body excluding palms and soles. The most likely
A. Measles diagnosis is:
B. Herpes A. Scarlet fever
C. Rubella B. Malaria
D. Streptococcal scarlet fever C. Dengue
D. Yellow Fever
KEY: C
KEY: D
2. Reference protein is
A. Milk protein 9. There is an epidemic of Meningococcal Meningitis among
B. Red mutton jail prisoners. The best chemoprophylaxis for the protection
C. Egg protein of contacts is by giving:
D. Fish protein A. Rifampicin
B. Chloramphenicol
KEY: C C. Doxycycline
D. Penicillin
3. Which of the following infection confers lifelong immunity?
A. Diphtheria KEY: A
B. Mumps
C. Typhoid 10. Six of the ten family members living in a single room house
D. Tetanus complain of intense itching with scratching in axillae, groin
& hands; it is more marked at night. The most likely
KEY: B diagnosis is:
A. Scabies
4. Kaplan-meier curve indicates B. Dermatitis
A. Incidence of disease C. Eczema
B. Demography D. Psoriasis
C. Severity of disease
D. Survival KEY: A

KEY: D 11. A 5 years old boy developed fever with typical “dew drop
rash” over his body. It appeared first on the trunk and
5. Vitamin D requirement for children: spread to arms and legs later; there were no signs of neck
A. 200 IU stiffness and rigidity. What could be likely diagnosis?
B. 100 IU A. Small pox
C. 400 IU B. Measles
D. 300 IU C. Meningococcal meningitis
D. Chicken pox
KEY: C
KEY: D
6. Varicella virus is:
A. Retrovirus 12. A school child is diagnosed to have chicken pox. He should
B. Herpes virus be isolated from other school children till: (NUMS PMDC
C. Pox virus 2019)
D. Entero virus A. The scabs fall off
B. Three days after the fever develops
KEY: B C. Five days after the development of pustules
D. Seven days after the development of pustules
7. A 22 years old man presented with painful, vesicular and
postulate eruption in the distribution of sensory nerve roots KEY: A

360
13. A mother brought her 4 years old child with complaint of 18. If you being a field doctor in department of public health,
sore throat, difficulty in swallowing and low grade fever. are given a task to visit a low socioeconomic community of
On examination mild erythema and whitish membrane was a slum and to give report about the immunization status of
found on the posterior pharynx. The doctor diagnosed him the community against Tuberculosis. The single most
as a case of Diphtheria Mother gave history of complete important clue to this immunization is
course of immunization. In addition to penicillin what A. Monteux test
would be your line of management? B. BCG scar
A. Supportive treatment C. X-ray chest
B. Passive immunization D. Sputum for AFB
C. Active immunization
D. Active plus passive immunization KEY: B

KEY: B 19. An epidemiologist was assigned to find out all the cases,
both new and old of T.B, in a slum located near Islamabad
14. An anxious mother came to OPD. Her son was diagnosed as during year 2007.Prevelance of tuberculosis is confirmed by:
a case of Meningitis last week. She was worried about her A. Mass miniature radiography
two daughters who were still well. Physician advised B. Sputum examination
chemoprophylaxis by: (NUMS PMDC 2019) C. Sputum culture
A. Ciprofloxacin D. Tuberculin test
B. Rifampicin
C. Penicillin KEY: C
D. Gentamicin
20. Ministry of health in collaboration with WHO is working to
KEY: B eradicate polio from Pakistan since 1992. But still there are
few cases of polio identified each year. Source of most
15. In Pediatric OPD the physician examined a 3 years old child Polio infection:
with low grade fever, mild erythema in the throat and A. Clinical carriers
whitish membrane on the left side tonsil. The cervical B. Healthy carriers
lymph node was palpable. The doctor advised the mother to C. Convalescent carriers
isolate the child for 7 days from other contacts of less than 5 D. Animal source
years old. The most probable diagnosis is
A. Pharyngitis KEY: C
B. Tonsillitis
C. Diphtheria 21. A 10 years old boy was brought to a doctor in a hospital
D. Whooping cough with history of moderate fever with shivering and abundant
rash on the trunk and buccal mucosa. On examination there
KEY: C were vesicles filled with clear fluid on the trunk and legs.
The physician told the mother that child is suffering from
16. A student of nursery class developed mild fever along with A. Herpes
irritating cough gradually becoming paroxysmal along with B. Chicken pox
characteristic whoop. What you suggest for how long the C. Rubella
student should be isolated from the class? D. German measles
A. I week
B. 2 weeks KEY: B
C. 3 weeks
D. 4 weeks 22. A man living in a slum area was brought to the emergency
of the DHQ Rawalpindi with history of passing rice water
KEY: D stools about 20-30 times a day and vomiting for last 1 day.
On examination there were signs of severe dehydration and
17. A 30 years old man presented in emergency in POF hospital patient was in shock. What is the likely diagnosis?
with complaint of severe headache, fever and vomiting. On A. Typhoid
examination neck stiffness was found. He has just returned B. Amoebic dysentery
from hajj and gives no history of preceding ailment or C. Cholera
injury. The most probable diagnosis is D. Food poisoning
A. Meningitis
B. Tetanus KEY: C
C. Brain abscess
D. Cerebral Malaria 23. In a shanty town of Karachi where intermittent water supply
system existing a woman noticed lots of mosquitoes which
KEY: A were black and spotted around the water tank. Keeping in
mind this situation which disease is likely to be seen in this
family?

361
A. Dengue KEY: C
B. Malaria
C. Filariasis 29. A 7 years old child presented with sore throat low grade
D. Encephalitis fever rash on face and posterior auricular and cervical
lymphadenopathy. The most probable diagnosis is
KEY: A A. Chicken pox
B. Measles
24. A mother brought her infant to a doctor and told him about C. Rubella
the severe itching and restlessness at night. Examination D. Mumps
revealed presence of burrows and vesicles especially on
sides of fingers and finger webs KEY: C
A. Scabies
B. Insect bite 30. You are performing the duties of Airport Medical Officer.
C. Prickly heat An Aircraft has landed from Africa and you found that one
D. Onchocerciasis of the passengers is not having the vaccination certificate
against yellow fever. What would you suggest for him?
KEY: A A. Vaccination
B. Quarantine for 10 days
25. After an epidemic of measles in a village near Taxila, some C. Isolation for 1 month
children reported with history of weight loss and blindness, D. Chemoprophylaxis
from corneal scarring. The most likely cause of this
complication is deficiency of KEY: B
A. Vitamin K
B. Vitamin D 31. A neonate presented in pediatric emergency on 8th day of
C. Vitamin A birth with high grade fever, locked jaw and stiffness of
D. Vitamin B12 whole body. Mother gave history of home delivery in a
village and application of cow dung on the umbilical stump.
KEY: C She gave no history of T.T during pregnancy. The most
probable diagnosis is:
26. An outbreak of measles occurred in rural areas surrounding A. Encephalitis
the District Rawalpindi. The local Government planned B. Meningitis
mass immunization of children against measles. Immunity C. Tetanus neonatarum
conferred by measles vaccine last for D. Epilepsy
A. 5 years
B. 10 years KEY: C
C. 15 years
D. Life long 32. A 25 years lady was brought in gynae emergency of PIMS
hospital at Islamabad with high grade fever, stiffness of
KEY: D whole body and locked jaw. Her mother gave the history
that she got an abortion done at 8th week from a local Dai 3
27. In a day care centre a child was having measles. In the days back. The per-vaginal examination revealed foul
centre the children between the ages of 9-12 months can be smelling purulent discharge. What is the most probable
protected against measles with measles vaccine, provided diagnosis?
that this is given with in A. Puerperal tetanus
A. 1 day of exposure B. PID
B. 3 days of exposure C. UTI
C. 7 days of exposure D. Vaginal diphtheria
D. 10 days of exposure
KEY: A
KEY: B
33. A 3rd year MBBS student of FJMC presented in ENT OPD
28. A pregnant lady reported to ante natal clinic with signs and with complaint of sore throat, pain of swallowing & low
symptoms of Rubella during 28th week of gestation. She grade fever examination revealed erythema on the pharynx
was insisting for the induction of labor because of fear of and a whitish membrane on the pharynx extending to the
congenital malformations of fetus. She was told by the left tonsil. The doctor diagnosed her as a case of diphtheria.
doctor that Rubella does not cause major abnormalities of What do you suggest minimum isolation period:
fetus after A. Six daily negative throat and nasal swabs report
A. 8th week of pregnancy B. Till the signs & symptoms settle down
B. 12th week of pregnancy C. One week course of antibiotic
C. 16th week of pregnancy D. Till complete blood picture becomes normal
D. 24th week of pregnancy
KEY: A

362
A. No treatment
34. A disease outbreak occurred in India last year. Many B. Isolation only
patients either presented with enlarged painful lymph nodes C. Quinolones for 1 week
or pneumonia with high grade fever and toxicity. High D. Ampicillin plus Probenacid for 1 week
mortality due to pneumonia was also reported. The likely
epidemic is: KEY: D
A. Dengue
B. Plague 40. A 22 years old married non-pregnant woman developed
C. Filariasis rubella infection. In order to avoid congenital rubella
D. Malaria syndrome in her pregnancy she should be given:
A. Antibiotics
KEY: B B. Active immunization
C. Nonspecific immunization
35. A mother brought her child with history of paroxysmal D. Advice to avoid conception for 12 weeks
cough and restlessness. On examination he showed a loud
inspiratory sound and sub-conjuctival haemorrhagic On the KEY: D
basis of clinical presentation what should be the drug of
choice: 41. A 40 years old man was diagnosed as a case of TB 4 weeks
A. Erythromycin ago. He has been taking ATT for the last 3 weeks. His
B. Ampicillin sputum analysis showed AFB on follow up investigation.
C. Tetracycline Such a case of TB is known as:
D. Co- trimoxazole A. Failure case
B. Newer case
KEY: A C. Defaulter
D. Resistant
36. A 40 year old tuberculosis patient on ATT for the last two
months presented to his physician with complaints of KEY: B
tingling, numbness and loss of peripheral sensation. The
likely anti-tuberculosis drug to have caused these symptoms 42. A primigravida presented in Medical OPD at 39 weeks of
is: gestation with dew drop rash on the body for 1 day she was
A. Isoniazid diagnosed as having chicken pox she was told that her baby
B. Rifampicin is at higher risk of having:
C. Pyrazinamide A. Low birth weight
D. Ethambutol B. Microcephaly
C. Atrophied limbs
KEY: A D. Varicella infections

37. 10 years old boy presented with high grade fever, chills, KEY: D
aches, cough and generalized weakness. He was diagnosed
as a case of influenza. The most dreaded complication is: 43. A 17 year old boy was brought in emergency department
A. Encephalitis with symptoms of acute encephalopathy. He was admitted
B. Pneumonia in ICU, initial investigation revealed that his liver had
C. Toxic shock syndrome undergone fatly degeneration. His father gave history of
D. Reye’s syndrome rash on his body. Most likely he suffered from:
A. Measles
KEY: B B. Rubella
C. Chicken pox
38. A patient comes to medical OPD with complaints of D. Cutaneous diphtheria
continuous fever and abdominal pain for ten days. His blood
culture shows S. typhi. The doctor decided to admit him in KEY: C
medical ward and the patient should be isolated till:
A. Six stool cultures become negative 44. After serial sonography it was told to the apparently healthy
B. Three stool culture become negative pregnant woman that her baby is microcephalic and of low
C. Three blood culture become negative birth weight she gave no important medical history of note
D. Patient become afebrile except mild febrile illness with rapidly disappearing rash in
2nd month of pregnancy. Most likely she suffered from:
KEY: B A. Chicken pox
B. German measles
39. On routine investigation of mess workers of girls’ hostel, Dr. C. Measles
Sarah noted that two of the mess worker showed positive D. Malaria
stool culture for S .typhi although they were symptom free.
They require: KEY: B

363
B. Circulatory collapse
45. 4 years old girl was having fever, cough with a C. Renal failure
characteristic whoop. She was diagnosed as a case of D. Sepsis
whooping cough several antibodies are effective they are
important as they: KEY: B
A. Reduce the frequency of spasm
B. Control severity of disease 51. 45 Years man was brought to emergency department of
C. Control secondary bacterial infection hospital with history of snake bit there was intense local
D. Prevent carrier state pain, swelling and ecchymosis at site of bit few hours later
bleeding started from the gums, followed by coma and
KEY: C death. The type of snake involved is:
A. Sea snake
46. 10 years old boy was brought to a rural health centre with B. Common krait
probable history of snake bite 12 hours back. On C. Green pit viper
examination of the wound, multiple small punctured lesions D. Elapid snake
with a mild swelling were observed; no other local &
systemic signs of envenomation were found, but the patient KEY: C
was very anxious. The management for this emergency is:
A. Polyvalent anti-snake venom serum
B. Reassurance 52. Six of the ten family members living in a single room house
C. Antibiotics complain of intense itching with scratching in axillae, groin
D. Tourniquet & bands; it is more marked at night. The most likely
diagnosis is:
KEY: B A. Scabies
B. Dermatitis
47. A 30 years old lady is bitten by a snake. She complains of C. Eczema
giddiness, lethargy, muscular weakness and spreading D. Psoriasis
paralysis. The type of snake involved is:
A. Sea snake KEY: A
B. Green pit viper
C. Elapid snake 53. Six of the ten family members living in a single room house
D. Russell’s viper complain of intense itching with scratching in axillae, groin
and hands; it is more marked at night. The most likely
KEY: C diagnosis is:
A. Scabies
48. A 7 years old child reported to a private hospital with B. Dermatitis
history of snake bite. On examination there were petechial C. Eczema
hemorrhages and bleeding from rectum. The toxic principal D. Psoriasis
in snake venom responsible for those signs is:
A. Proteolysin KEY: A
B. Neurotoxin
C. Cholinesterase 54. In a house consisting of two living rooms, the door and
D. Thromboplastin windows are facing each other. This will provide:
A. Low humidity
KEY: D B. Aspiration
C. Cross ventilation
49. A case of snake bite was brought to a basic health unit. D. Diffusion
According to the American Red Cross the immediate step to
be taken is: KEY: C
A. Ice packing
B. Apply tourniquet 55. A 12 member’s family was living in a house consisting of
C. Give incisions in the wound two rooms. Which disease is most likely to be common in
D. Immobilize the bitten area the given situation?
A. Asthma
KEY: B. Tuberculosis
C. Cystic fibrosis
50. 45 Years old man was brought to emergency department of D. Emphysema
hospital with history of snake bit. There was intense local
pain, swelling and ecchymosis at site of bit few hours later KEY: B
bleeding started from the gums, followed by coma and
death. The most probable cause of death is:
A. Respiratory paralysis

364
56. A 5 member family was residing in a small house. The
available floor space to one person was 30 sq. ft. The 62. A lady brought her 8 years old child to a doctor. She
problem which is more likely to be associated with this complained that her child was unable to see things properly
available space is: at night: child had been having diarrhoea off and on. Which
A. Psychosocial vitamin deficiency is most likely in this child?
B. Malnutrition A. vitamin A
C. Enterobius vermicularis B. Vitamin D
D. Euphoria C. vitamin E
D. vitamin K
KEY: A
KEY: A
57. If a child presents with protein energy malnutrition showing
signs of loss of subcutaneous fat and weight reduction. The
level of prevention suggested at this point is: 63. 2 years old child was brought to the emergency department.
A. Primordial prevention He was having convulsions. On a rapid general physical
B. Specific protection examination kyphoscoliosis was discovered X-ray showed
C. Early diagnosis and prompt treatment swollen lower end of radius. What is the likely diagnosis?
D. Disability limitation and rehabilitation A. Osteomalacia
B. Rickets
KEY: D C. Pellagra
D. Beriberi
58. A child of 1 year presents with muscle wasting, loss of
subcutaneous fat with no signs of edema and weight below KEY: B
60% of WHO standard. The mother gives history of not
giving enough proteins and other nutrients to the child after
six months of age. The likely diagnosis is: 64. A Pakistani physician went to work in South Africa. A
A. Kwashiorkor woman reported with diarrhoea. On examination she was
B. Marasmus found to have glossitis and stomatitis. Her detailed
C. Under nutrition investigations revealed presence of anemia. She was
D. Vitamin B1 deficiency diagnosed as a case of Niacin deficiency. The likely food to
have caused this deficiency state is:
KEY: B A. Legumes
B. Maize
59. A weight conscious pregnant woman wants information C. Whole wheat
about her requirement of calories per day during pregnancy. D. Raw rice
You suggest an increase of:
A. 450 kcal KEY: B
B. 350 kcal
C. 550 kcal
D. 650 kcal 65. A chronic alcoholic was complaining of loss of appetite and
pain in lower legs. On examination his gate was ataxia. He
KEY: B is suffering from deficiency of:
A. Niacin
60. A 5 years old child complains of poor vision at night with B. Thiamine
no other refractive error. He is likely to be benefited by: C. Folate
A. Cod liver oil capsules D. Pantothenic acid
B. Eye drops containing antibiotics
C. Suitable eyeglasses KEY: B
D. Intra ocular lens replacement
66. A patient of pulmonary tuberculosis was put on anti-
KEY: A tuberculosis therapy. He was given Rifampicin, INH,
Ethambutol and Pyrazinamide He should be advised to have
61. A strict vegetarian, Rajesh 23 year’s old foreign student in a supplement of:
WMC is having lethargy, easy fatigue and palpitations. He A. Niacin
is found to have macrocytic anemia on CBC. He is suffering B. Pyridoxine
from deficiency of vitamin: C. Riboflavin
A. Vitamin B1 D. Thiamine
B. Vitamin B6
C. Vitamin B12 KEY: B
D. Vitamin D
67. A pregnant lady comes to Gynae OPD complaining about
KEY: C increasing lethargy and shortness of breath. On examination

365
she is found to have glossitis. Her CBC reveals macrocytic A. Chloride
anemia. The most likely deficiency which has caused this B. Calcium
condition is: C. Fluoride
A. Folic acid D. Zinc
B. Vitamin B6
C. Niacin KEY: C
D. Thiamine
73. The non-clinical Vit-A deficiency is more common as
KEY: A compared to the clinical, threatening the health of as many
as one third of the world’s children. The best proxy
68. A 30 years old lady was brought to the hospital with strong indicator of this is:
labour pains for the last 3 hours and no progress. She was A. Infant mortality rate
found to have pelvic deformities which lead to B. Maternal mortality rate
cephalopelvic disproportion. The baby was delivered by a C. Literacy rate
cesarean section. The woman is likely to have suffered from D. Specialized medical care
the deficiency of:
A. Vitamin A KEY: A
B. Vitamin C
C. Vitamin D 74. A 4 year old child was brought in Pediatric OPD with
D. Vitamin K complaint of inability to see at night for the last few days,
Pediatrician suspected deficiency of vitamin. Although the
KEY: C clinical signs of vitamin A deficiency are rare, but if
occurred, what would be the first clinical sign?
69. A person who subsisted on taking maize only for many A. Conjuctival xerosis
years reported to a health facility with the signs of glossitis. B. Bitot spot
His history suggested frequent attacks of diarrhea and C. Corneal xerosis
memory loss. The likely condition is: D. Keratomalacia
A. Pellagra
B. Beri-Beri KEY: A
C. Iron deficiency
D. Protein malnutrition 75. In a Madrasa of a remote area, 14 girls of 9 years of age
were residing to memorize Quran Pak. They were treated
KEY: A well but they observed strict Purdah and were never allowed
to visit outside. On returning home after 5 years, 10 of them
were short stature. The most likely reason of being short
70. Strictly vegetarian Hindu teacher presented with lower statured:
limbs weakness. His examination revealed upper motor A. Vitamin B12
neuron signs in lower limbs. His blood CP showed B. Vitamin C
macrocytic anemia. He is probably suffering from C. Vitamin D
deficiency of vitamin: (NUMS PMDC 2019) D. Vitamin K
A. Vit-B6
B. Vit-B12 KEY: C
C. Vit-B1
D. Vit-D 76. Researches decided to study the impact of iodized salt
programme especially in hilly areas of Pakistan. They had
KEY: B chosen the most sensitive indicator for monitoring
environmental iodine deficiency which was:
71. 15 years old boy living in Murree is having swelling in the A. Prevalence of goiter
neck, Apart from weakness and constipation; there are no B. Prevalence of myxedema
other complaints at present. He is likely to be suffering from C. Prevalence of neonatal hypothyroidism
deficiency of: D. Urinary iodine excretion
A. Calcium
B. Iodine KEY: C
C. Iron
D. Sodium

KEY: B 77. Government of Pakistan has accepted iron fortification to


reduce prevalence of anemia; it has recently been decided to
72. A dental surgeon appointed in a rural health centre reports fortify:
an increased incidence of dental caries in the people of that A. Salt
area. Research team confirmed that water supply of that B. Flour
area is deficient in: C. Sugar

366
D. Skimmed dried milk KEY: C

KEY: B 83. 40 years old Bank officer was told to be having illeocecal
Tuberculosis (Bovine after thorough investigation. He told
78. Flourosis develops when water fluoride content is above the physician that he had no body infected with tuberculosis
15mg/lit but it has been observed after different studies that in the family, but he was told by the Doctor that he acquired
this flourosis can also develop in people whose staple diet is: this infection from:
A. Wheat A. Infected milk
B. Rice B. Infected client
C. Jowar C. Contaminated vessel
D. Maize D. Polluted water

KEY: C KEY: A

79. At 1:00 pm a mother got a phone call from the school to 84. An obese lady accountant working in POF hospital Wah
pick her son who was a student of 3 class, as he was having Cantt came to the dietician there and took advice for a
excessive vomiting. The child was taken immediately to the balanced diet. Dietician told that important point for prudent
Hospital where he admitted that he had taken ice-cream in diet is that:
lunch break at 11:30 am. Most likely he was suffering from A. Dietary fat should be limited to 20-30% of total intake
food poisoning due to: B. Unsaturated fats should be less than 10%
A. Salmonella C. Avoidance of complex carbohydrates
B. Shigella D. Proteins should account for 50-60% of diet
C. Staph aureus
D. Vibrio cholera KEY: A

KEY: C 85. A man belonging to a poor community presents with


diarrhea and dermatitis. He also shows signs of personality
80. Researchers surveyed the causes of vitamin D deficiency and memory dysfunction. His history suggests that his
and its geographic distribution. They found that exposure to staple diet is maize. The likely diagnosis is:
ultra violet rays is crucial for its adequacy, which when get A. Pellagra
excessively filtered by the skin result in its deficiency. This B. Vitamin B12 deficiency
deficiency is more common in: C. Riboflavin deficiency
A. Asians D. Biotin deficiency
B. Black Africans
C. Europeans KEY: A
D. Americans
86. Riboflavin is an important group of water soluble vitamin.
KEY: C Cereals and pulses are relatively good sources of Riboflavin.
Most common lesion associated with Riboflavin, which can
also be used as an index of state of nutrition of group of
81. 38 years old man looking Jaundiced presented in children is:
emergency with bruises and hematuria. History revealed A. Follicular keratosis
that he was having cholestatic Jaundice leading to vitamin K B. Angular stomatitis
deficiency which had ultimately resulted in markedly C. Dementia
decreased: D. Dermatitis
A. Prothrombin
B. Serotonin KEY: B
C. Arachidonic acid
D. Platelet derived growth factors 87. A four year old child presented in an OPD with signs of
edema on limbs, blond sparse hair and dermatosis. His
KEY: A weight was 70% of the standard for his age. The likely
condition is:
82. District health officer visited local general stores in A. Marasmus
Gujranwala District. He found certain substandard products, B. Seborrhoeic dermatitis
Turmeric was found to be containing lead chromate powder C. Wet beriberi
and Coriander, cow dung. He reported to the health D. Kwashiorkor
authorities that the foods in Gujranwala district are:
A. Intoxicated KEY: D
B. Fortified
C. Adulterated 88. A chronic alcoholic was complaining of loss of appetite
D. Infected and pain in lower legs. On examination his gate was ataxia.
He is suffering from deficiency of: (NUMS PMDC 2019)

367
A. Niacin impair an individual’s ability to function effectively and
B. Thiamine may result in physical, social and
C. Riboflavin emotional harm is best defined as:
D. Folate A. Drug abuse
B. Drug dependence
KEY: B C. Drug tolerance
D. Drug addiction
89. A mother brought her six years old child to Eye OPD with
history of night blindness. She told that the baby suffered KEY: A
from measles 6 months ago. Examination revealed corneal
scarring. The doctor should suspect deficiency of: 95. A 30 years old man presented in urology OPD with
A. Vitamin C complains of haematuria at the end of micturition not
B. Vitamin A associated with pain, urgency or frequency urine R/E was
C. Vitamin D suggested of haematuria. Microscopic examination of the
D. Vitamin E urine revealed presence of oval shaped eggs. Most probably
he is suffering form:
KEY: B A. Vesical calculus
B. Carcinoma bladder
90. Riboflavin is an important group of water soluble vitamin. C. Urinary schistosomiasis
Cereals and pulses are relatively good sources of Riboflavin. D. Trichomonas vaginalis infections
Most common lesion associated with Riboflavin deficiency,
which can also be used as an index of state of nutrition of KEY: C
group of children is:
A. Cheilosis 96. The only reservoir of Ascaris lumbricoides (round worm) is:
B. Glossitis A. Dogs
C. Angular stomatitis B. Cattle
D. Nosolabial dyssberia C. Man
D. Birds
KEY: C
KEY: C
91. 45 years old hypertensive bank manager is used to high salt
intake in food. Which type of salt would be less harmful for 97. In prevention of traffic accidents the most effective measure
this man? is: (NUMS PMDC 2019)
A. Fluoride A. Licensing of drivers
B. Potassium
B. Provision of seat belts
C. Chromium
D. Manganese C. Enforcement of traffic laws
D. Inspection of vehicles periodically
KEY: B
KEY: C
92. Use of oral contraceptives by women smokers over 35 years
of age is associated with increased risk of: 98. The approach of rapidly classifying the injured on the basis
A. Cervical cancer of severity of their injuries and likelihood of their survival
B. Breast carcinoma with prompt medical intervention after disaster is called as:
C. Chronic bronchitis ( NUMS PMDC 2018-19)
D. Coronary heart disease A. Search, rescue and first aid
B. Field care
KEY: D C. Tagging
D. Triage
93. A 40 years old woman taking oral anti-diabetics for last 5
years, with poor control of blood sugar presented to KEY: D
physician for routine checkup. The test which provides a
long term index of glucose control in this case would be: 99. A mother noticed that her 3 year old boy recently admitted
A. Standard oral glucose test in play group class at Lahore grammar school was
B. Glycosylated hemoglobin (HbA1c) scratching his scalp for the last one week. While combing
C. Urine sugar examination his hair she noticed small white ovoid nits. The most
D. Fasting blood sugar examination effective treatment for this condition is use of topical
solution containing: (NUMS PMDC 2019)
KEY: B A. 0.05 % Malathion
B. DDT
94. Self-administration of drugs for non-medical reason in C. Permethrin
frequency and quantities that may D. Mineral oil

368
D. Silicosis
KEY: A
KEY: D
100. The total number of people in a completed family can be
estimated from: (NUMS PMDC 2019) 106. A shipyard worker presents with increasing breathlessness.
A. Net reproduction rate His X-ray shows ground glass appearance in lower two thirds
B. Gross reproduction rate of lungs. He is likely to be suffering from: (NUMS PMDC
C. Contraception prevalence rate 2017)
D. Total fertility rate A. Anthracosis
B. Silicosis
KEY: D C. Asbestosis
D. Byssinosis
101. The number of daughters a new born girl will bear during
her life time assuming fixed age specific fertility and KEY: D
mortality rate, refers to which one of the following? (NUMS
PMDC 2019) 107. An occupational worker presented with cause complaints of
A. Age specific fertility rate exertional dyspnea. He gave history of being in an industry
B. Gross reproduction rate dealing with spare parts such as gas kit and brakes, he also
C. Net reproduction rate gave history of smoking for about five years, His x-ray
D. Total fertility rate chest showed a ground glass appearance/ honey combing in
the lower thirds of the lung fields. The likely condition that
KEY: C he suffers from is: (NUMS PMDC STEP 2018)
A. Anthracosis
102. Demographic cycle has five stages and each country is B. Asbestosis
allotted a different stage according to its distribution of C. Siderosis
population. For examples if the death and birth rate of a D. Silicosis
country both are declining then we call it as:
A. High stationary KEY: B
B. Early expanding
C. Late expanding 108. A 5 year old child comes to the immunization centre
D. Low stationary without BCG scar on his arm. It is advisable to give:
A. BCG vaccine
KEY: C B. Chemoprophylaxis
C. BCG if Mantoux test positive
103. 45 Years old man was brought to emergency department of D. BCG if Mantoux test negative
hospital with history of snake bite. There was intense local
pain, swelling and ecchymosis at site of bite. Few hours KEY: D
later bleeding started from the gums, followed by coma and
death. The type of snake involved is 109. A woman reports for vaccination against Tetanus only 25
A. Sea snake days before delivery; she has not
B. Common krait received the first dose. What will you give?
C. Green pit viper A. ATIG at present
D. Cobra B. Two doses of T.T ,2 weeks apart
C. Antibiotic during delivery
KEY: D D. ATIG after delivery

104. A 30 years old lady is bitten by a snake. She complains of KEY: D


giddiness, lethargy, muscular weakness and spreading
paralysis. The type of snake involved is: 1. 10 cases of food poisoning had been reported in hospital, 2
A. Sea snake out of these developed mild gastrointestinal symptoms, 4
B. Green pit viper developed moderate dehydration but recovered and 2
C. Elapid snake succumbed to the disease. The characteristic of the
D. Viper snake organism of food poisoning that produces the severest form
of the disease is — Virulence
KEY: C A. Infectivity
B. Pathogenicity
105. Pottery industry worker developed Tuberculosis. The likely C. Virulence
condition which made him prone to tuberculosis was: D. Communicability
(NUMS PMDC 2017)
A. Anthracosis KEY: C
B. Asbestosis
C. Begassosis

369
2. District health officer visited local general stores in D. Case fatality rate
Gujranwala District. He found certain substandard products,
Turmeric was found to be containing lead chromate powder KEY: D
and Coriander, cow dung. He reported to the health
authorities that the foods in Gujranwala district are 8. Japanese have a higher rate of stomach cancer and a low
A. Intoxicated rate for colon carcinoma than theU.S. However third
B. Fortified generation descendants of Japanese immigrants to U.S have
C. Adulterated rates of stomach colon cancer like that of U.S. This
D. Infected particular characteristic supports effects of
A. Environment
KEY: C B. Genetics
C. Mutation
3. 40 years old Bank officer was told to be having illeocecal D. Misinterpretation
Tuberculosis (Bovine after thorough investigation. He told
the physician that he had no body infected with tuberculosis KEY: A
in the family, but he was told by the Doctor that he acquired
this infection from 9. Influenza pandemic occurs after every 7 – 10 years. This
A. Infected milk kind of disease distribution in time is known as
B. Infected client A. Secular trend
C. Polluted water B. Short time fluctuation
D. Intake of raw vegetables C. Cyclical trend
D. Seasonal trend
KEY: A
KEY: C
4. An obese lady accountant working in POF hospital Wah
Cantt came to the dietician there and took advice for a 10. If the age incidences curve of leukemia shows two peaks it
balanced diet. Dietician told that important point for prudent is suggestive of bimodality.Bimodality usually signifies
diet is that A. Non-homogeneity
A. Dietary fat should be limited to 20-30% of total intake B. Cluster sampling
B. Unsaturated fats should be less than 10% C. Large number of observations
C. Avoidance of complex carbohydrates D. Short duration of disease
D. Proteins should account for 50-60% of diet
KEY: A
KEY: A
11. A doctor is required to study the incidence of silicosis in a
5. Public Policies in Pakistan aim at avoiding the underlying stone cutting industry, which study design should he choose
reasons for the development of environmental and A. Longitudinal
atmospheric concentration of SO2 to protect the health of B. Cross-sectional
people, it’s an example of C. Ecological surveys
A. Primordial D. Case series report
B. Primary
C. Secondary KEY: A
D. Screening
12. The health statistics department revealed that the sale of
KEY: A anti-Asthma drugs was more in those countries where
Asthma deaths were more. This association may prove
6. The number of deaths due to diarrhoea, total cases of wrong when the individual based study designs are
measles, total number of accidents and the total number of conducted. This association is an example of
drug addicts were to be reported by a researcher. The best A. Ecological fallacy
title given to all of this data would be B. Berkesonian bias
A. Mortality data C. Indirect association
B. Morbidity data D. Specific association
C. Case fatality rate
D. Health related data KEY: A

KEY: D 13. A researcher wanted to study the time sequence to prove the
concept of causatively, which design of study should be
7. When total number of deaths due to measles is presented in preferred by the researcher
relation to the total cases of measles, it is best labeled as A. Longitudinal
A. Cause specific death rate B. Cross-sectional
B. Incidence rate C. Case report
C. Prevalence rate D. Case series report

370
B. Rate
KEY: A C. Ratio
D. Frequency
14. Smoking leads to esophageal carcinoma Coffee intake has
its effect on smoking and also esophageal carcinoma. This KEY: A
factor can distort the results of the study which intends to
prove an association between smoking and esophageal 20. The trend in mortality from tuberculosis in England showed
cancer. This effect of this factor is known as a steady fall in years 1855 – 1965 but thereafter a gradual
A. Confounding rise in the incidence of this disease was reported. This type
B. Multiple causation of time trend or fluctuation in disease occurrence is termed
C. One to one relationship as
D. Dose response relation A. Epidemic trend
B. Cyclical trend
KEY: A C. Seasonal trend
D. Secular trend
15. The health authorities are launching a smoking cessation
program by designing different activities for the smokers. KEY: D
These are very expensive but still useful as a large
proportion of lung cancer will be eliminated if smoking is 21. In the mid nineteenth century, an epidemiologist suggested
stopped. This proportion of lung cancer can be indicated by that cholera was caused by drinking water in which an
A. Relative risk invisible agent is present. This type of association gives
B. Prevalence A. Specificity
C. Attributable risk B. Temporal sequence
D. Attributable fraction C. Biological plausibility
D. Consistency
KEY: D
KEY: C
16. A researcher was studying maternal mortality in Rawalpindi
District. He observed more deaths in women who were 22. The incidence of pollen allergy at Wah Cantt is 10 cases per
brought to hospital and without taking other factors into thousand populations. The mean duration of illness is 3
account concluded that hospital managed cases have more months from February to April. The prevalence of pollen
mortality as compared to home deliveries. This is an allergy at Wah Cantt is
example of A. 10
A. Indirect association B. 20
B. Relative risk C. 30
C. Spurious association D. 40
D. Attributable risk
KEY: C
KEY: C
23. When a new treatment is developed that delays deaths but
17. Early diagnosis and prompt treatment is focused on does not produce recovery from a chronic disease, which of
A. Disease identification the following will occur
B. Host factors A. Prevalence of the disease will decrease
C. Environmental factors B. Incidence of the disease will increase
D. Restoration of ability C. Prevalence of the disease will increase
D. Incidence of the disease will decrease
KEY: A
KEY: C
18. An expert in the field of public health is required to estimate
the magnitude of a health problem. Which rate would he 24. If the number of deaths from tuberculosis is expressed in
calculate for this—Prevalence relation to the total mid-year population, it is
A. Incidence A. Case fatality rate
B. Prevalence B. Age specific death rate
C. Case fatality C. Crude death rate
D. Proportionate mortality D. Cause specific death rate

KEY: B KEY: D

19. When the number of educated females is expressed as a 25. The proportion of the disease in a population that would be
percentage of total females present in a village. It is known eliminated if the risk factor is eliminated is determined by
as A. Relative risk
A. Proportion B. Absolute risk

371
C. Attributable fraction
D. Odds ratio KEY: B

KEY: C 31. Prophylactic administration of vitamin K in breast fed


babies is an example of
26. A 39-year-old man who presents with a mild sore throat, A. Health Promotion
fever, malaise, and headache is treated with penicillin for B. Treatment
presumed streptococcal infection. He returns after week C. Specific protection
with hypotension, fever, rash, and abdominal pain. He D. Rehabilitation
responds favorably to chloramphenicol, after a diagnosis of
Rocky Mountain spotted fever is made which option KEY: C
explains the given example
A. Case series report 32. In an epidemiological study the incidence of disease in
B. Case-control study females is more than that of males but the prevalence is
C. Cohort study equal in both. It indicates
D. Case report A. Case fatality is more in female
B. Mortality in male is higher
KEY: D C. Disease is of less duration in males
D. Females harbour disease for longer duration
27. A total of 3500 patients with thyroid cancer are identified
and surveyed by patient interviews regarding past exposure KEY:
to radiation. Which options explains the given example
A. Case series report 33. Bhopal gas tragedy is an example of
B. Case-control study A. Slow epidemic
C. Clinical trial B. Continuous epidemic
D. Cohort study C. Point source epidemic
D. Propagated epidemic
KEY: A
KEY: C
28. A total of 10,000 Vietnam veterans, half of whom are
known by combat records to have been in areas where 34. Residents of three villages with three different types of
Agent Orange was used and half of whom are known to water supply were asked to participate in a study to identify
have been in areas where no Agent Orange was used, are cholera carriers because several cholera deaths had occurred
asked to give a history of cancer since discharge. Which in the recent past. Virtually everyone was present at the time
option explains the given example of examination. The proportion of carriers in each village
A. Case series report was computed and compare. This study is a:
B. Case-control study A. Cross-sectional study
C. Clinical trial B. Case-control study
D. Cohort study C. Concurrent cohort study
D. Non-concurrent cohort study
KEY: D
KEY: A
29. Patients admitted for carcinoma of the stomach are age and
sex-matched, with smoking history to assess the possible 35. In a village of 1 lakh population, among 20,000 exposed to
association. Which option explains the given example smoking, 200 developed cancer, and among 40,000people
A. Case series report unexposed, 40 developed cancer. The relative risk of
B. Case-control study smoking in the development of cancer is— 10
C. Clinical trial A. 10
D. Cohort study B. 15
C. 20
KEY: B D. 25

30. A highway patrol statistics revealed that more accidents KEY: A


occurred to blue cars than to cars of any other colour. The
inference that, while driving a blue car, one is at higher risk 36. In a population of 1000, measles coverage is 60%, one child
of accident than while driving a car of another colour is: goes out of station and comes back with measles from
A. Correct whom 26 more children get the measles. The secondary
B. Incorrect, because the comparison is not based on rates attack rate is:
C. Incorrect, because no control or comparison group is A. 6.5
used B. 65
D. Incorrect, because no test of statistical significance has C. 7.5
been made D. 7.0

372
recovered but was unable to move his right lower limb. He
KEY: A was advised physiotherapy. This type of assistance is
A. Specific protection
37. To compare the death rate of India with the death rate of B. Prompt treatment
Pakistan, the most appropriate measure is a comparison C. Disability limitation
between D. Rehabilitation
A. Age specific mortality rates
B. Crude death rates Key: D
C. Maternal mortality rates
D. Standardized mortality rates 43. According to a study conducted by WHO the incidence of
polio in Sindh province having a total population of 20
KEY: A million was 1 during the year 2007, which type of study was
this
38. About 2500 deaths were reported in road side accidents A. Case report
during the year 2006 in Pakistan. If the total number of B. Cross sectional
deaths due to accidents is expressed against the mid-year C. Case control
population of Pakistan in year 2006 this will give D. Cohort
A. Crude death rate
B. Age specific death rate Key: D
C. Cause specific death rate
D. Case fatality rate 44. An epidemiologist is assigned to conduct a study on 5000
people having hyperlipidemia and those having normal lipid
KEY: C profile He has to keep track of all the participants to observe
the development of stroke in these patients to confirm that
39. In a universe comprising of 1500 children less than 5 years hyperlipidemia increases the risk of stroke. This study is
of age, 75 children with severe malnutrition were found. If A. Retrospective cohort study
75 new cases of severe malnutrition were registered over a B. Retrospective study
period of one year, the incidence rate for severe C. Prospective study
malnutrition during the same year is D. Cross – sectional study
A. 50 / 1000
B. 53 / 1000 Key: C
C. 55 / 1000
D. 60 / 100 45. To compare the death rate of Nepal with the death rate of
Pakistan, the most appropriate measure is a comparison
KEY: B between
A. Age specific mortality rates
40. Every year during the winter season the hospital admissions B. Crude death rates
are more for pneumonia cases. This year also about 358 C. Maternal mortality rates
children with pneumonia were admitted in the Pediatric D. Standardized mortality rates
department of CMH Hospital between the months of Jan –
Mar 2009. This increased frequency of respiratory Key: D
infections during winter months is an example of
A. Epidemic trend 46. A 40 years old man of 75 kg came to a physician for his
B. Cyclical trend routine checkup. His serum cholesterol was found to be 230
C. Seasonal trend mg/dL and he was diagnosed as hypertensive. The risk
D. Secular trend factor of this particular condition is classified as
A. Physical
Key: C B. Chemical
C. Biological
41. Prevalence measures the burden of disease in a population D. Mechanical
inclusive of old & new cases. Prevalence of a disease can be
obtained from— Cross – sectional study Key: B
A. Quasi – experimental study
B. Cross – sectional study 47. Acute hemorrhagic conjunctivitis affected a large
C. Case – control study proportion of population over a wide geographic area in
D. Cohort study 1971 and 1981. This spread of disease is
A. Epidemic
Key: B B. Sporadic
C. Pandemic
42. A 55 years old hypertensive patient was admitted in the D. Endemic
Medical Ward with cerebral stroke. After treatment he
Key: C

373
haziness is also seen. There is no history of any other
48. A patient came in emergency with signs of dehydration and disease or injury. The likely agent type is:
severe diarrhea. An Intra venous infusion was given to A. Physical and chemical
correct electrolytes and fluid levels. He was discharged after B. Nutritional
2 days. About 2 months later the patient came back with C. Hormonal
signs of jaundice and Hepatitis B surface antigen was D. Immunological
Positive. He did not give history of any event which could
have led to this disease This hepatitis Infection may be KEY: B
labeled as
A. Sub clinical 54. A man brought his child with complaints of fever, diarrhea
B. Opportunistic and abdominal pain. He gave history of living in an
C. Cross infection unhygienic small house around a factory area where his son
D. Iatrogenic had many friends. He had three more children who also had
same complaints off and on. Which environment is likely to
Key: D have resulted in this condition
A. Physical
49. A cross-sectional study was conducted at Wah Medical B. Chemical
College in the year 2006 to measure the period prevalence C. Psychological
of smokers among 105 students. Out of them 5 were already D. Occupational
smokers and 15 started during 2006, period prevalence of
2006 is: KEY: A
A. 22%
B. 19% 55. A new drug was introduced in some of patients to assess its
C. 15% usefulness compared with the old on patients nor did
D. 11% clinicians who evaluated patients for effect under
consideration in this clinical trial know individual treatment
Key: B assignments. This method of assignment is known as
A. Single blinding
50. In a village of population 10,000, 250 cases of Hepatitis B B. Double blinding
were reported in the month of July.The point prevalence of C. Triple blinding
Hepatitis B per thousand populations is D. Randomization
A. 20
B. 25 KEY: B
C. 30
D. 50 56. 10 cases of food poisoning had been reported in a hospital,
2 out of these developed mild gastrointestinal symptoms, 4
Key: B developed moderate dehydration but recovered and 2
succumbed to the disease. The characteristic of the
51. A household survey of 10 families was conducted by organism of food poisoning that produces the severest form
students of 4th year MBBS, Wah Medical College In the of the disease is
data they collected, the ages of heads of families were: 32, A. Infectivity
34, 35, 36, 36, 42, 44, 46, 48, and 52. The mean age of B. Pathogenicity
heads of families is C. Virulence
A. 36 D. Communicability
B. 38.5
C. 40 Key: C
D. 40.5
57. The trend in mortality from TB in England showed a steady
Key: D fall in years 1855 – 1965 but thereafter a gradual rise in the
incidence of this disease was reported. This type of time
52. A woman brings her child to the hospital for mongolism. trend or fluctuation in disease occurrence is termed as
The possible agent of the disease that comes in your mind is A. Epidemic trend
A. Bacteria/Virus B. Cyclical trend
B. Nutritional factor C. Seasonal trend
C. Hormonal factor D. Secular trend
D. Chromosomal factor
Key: D
Key: D
58. An outbreak of brucellosis in cattle is reported, threatening
53. A child of three years comes with complaints of night the health of human population. This outbreak is
blindness. On examination conjunctiva is dry and corneal A. Epizootic
B. Epornithic

374
C. Enzootic
D. Exotic 66. The single most important practice to reduce hospital
associated infections is:
Key: A A. Hand washing
B. Isolation of infectious disease patients.
59. Incubation Period refers to C. Reverse flow hospital rooms
A. Time between exposure and onset of symptoms D. Face masks
B. Time between appearance of symptoms and recovery
phase Key: A
C. Time between exposure and course of illness
D. Time between febrile phase and critical phase 67. The average incubation period for HAV is:
A. 28 days
Key: A B. 45 days
C. 120 days
60. Dengue hemorrhagic fever results from: D. 3 months
A. Dengue 4
B. Dengue 1 Key: A
C. A second infection with a different dengue strain
D. Dengue 3 68. The average incubation period for HBV is:
A. 28 days
Key: C B. 45 days
C. 120 days
61. Dengue fever incubation period ranges from: D. 3 months
A. 1-12 days
B. 3-14 days Key: A
C. 14-28 days
D. 28-32 days 69. Children should be given an HAV vaccine when they are:
A. In their second year of life
Key: B B. At 2-5 years
C. At 5-10 years
62. The vector for chickungunya virus is: D. In their first year of life
A. Culex tarsalis
B. Aedes Aegypti Key: A
C. Anopheles
D. Any mosquito 70. Globally, the leading viral cause of hepato-cellular
carcinoma is:
Key: A A. HAV
B. HBV
63. The vector Aedes Aegypti is responsible for C. HCV
A. Yellow fever D. HDV
B. Dengue fever
C. Malaria Key: B
D. Filariasis
71. Ebola virus epidemics occur in:
Key: B A. Central Africa
B. Southeast Asia
64. The greatest number of deaths globally caused by a parasite C. Central America
are due to: D. All of the above
A. Schistosomiasis
B. Malaria Key: A
C. Filariasis
D. Ascariasis 72. The categories of “transmission-based precautions” include:
A. Hand hygiene, face masks, gowns
Key: B B. Contact, hand hygiene, gowns
C. Droplet, contact, airborne
65. The highest risk for hospital-associated infections occurs in: D. Droplet, face masks, reverse flow rooms
A. Cancer units
B. Infectious disease units Key: B
C. Intensive care units
D. Metabolic disease units 73. Individuals who test negative for anti-HCV antibodies:
A. Are not infected with HCV
Key: C

375
B. Should be confirmed negative using the recombinant C. No physical symptoms
immunoblot assay D. All of the above
C. Should be confirmed negative using a nucleic acid test
D. Should be treated for subclinical HCV infection Key: B

Key: A 81. Treatment of acute diarrhea includes:


A. Oral rehydration and penicillin
74. An individual who is positive only for anti-HBc-IgG + anti- B. Oral rehydration, zinc supplementation
HBs is: C. Intravenous saline and penicillin
A. Acutely infected D. Oral rehydration, energy-rich food, and zinc
B. Chronically infected supplementation
C. Immune after recovering from infection
D. Not infected Immune because they have been Key: B
effectively vaccinated
82. A survey report in 1960 concluded that there was an
Key: C increase in asthma deaths with the increased use of
pressurized aerosol bronchodilators; although the deaths
75. Which of the following is the major vector carrying malaria? were more because of the severity of disease. This
A. Culex tarsalis association is
B. Aedes Aegypti A. Spurious
C. Anopheles B. Indirect
D. Ratfinkus C. Consistency
D. Coherence
Key: C
Key: A
76. Which of the following vaccine types provides the strongest
immune response to protectagainst an agent? 83. In a coal mine the expected deaths of coal worker were 7
A. Killed while the deaths that really occurred were 9. The
B. Live, attenuated standardized mortality ratio for coal workers is
C. Toxoid A. 100
D. Anti-toxin B. 109
C. 11
Key: B D. 5+

77. A vaccine adjuvant Key: D


A. Increases the magnitude of the immune response
B. Increases the specificity of the vaccine 84. At Lahore Grammar School a student of class II developed
C. Assures a T cell response mumps. He was isolated from other children till swelling
D. All of the above subsided and his brother of class IV who looked apparently
healthy was also advised to be away from school for about a
Key: A fortnight. His brother’s type of carrier state is most likely to
be
78. Which of the following vaccine types can revert to virulence? A. Incubatory
A. Killed B. Healthy
B. Live attenuated C. Convalescent
C. Recombinant D. Temporary
D. Polysaccharide
Key: A
Key: B

79. The HPV strains that cause the majority of cervical cancer 85. A colony located near an industrial area 50 people died due
are: to asphyxia and many developed difficulty in breathing and
A. 16 and 18 were hospitalized within 24 hours. The cause was the
B. 6 and 8 leakage of carbon monoxide from a nearby chemical plant.
C. 6 and 11 The distribution of cases in time is suggestive of
D. 11 and 18 A. Propagated epidemic
B. Slow epidemic
Key: A C. Common source – single exposure
D. Common source – continuous exposure
80. A TB patient is considered non-infectious when he/she has:
A. A negative chest x-ray Key: C
B. Three sequential negative sputum smears

376
86. Cement industry is suspected for more deaths among its B. Between 7400 and 7800
workers. So the industrialist gets worried and wants to C. Below 6200 or Above 9000
assess whether more deaths are likely in these workers or D. Below 7600
not. The measure that predicts the mortality in this industrial
group is—Standardized mortality ratio Key: D
A. Age specific death rate
B. Standardized mortality ratio 92. Higher crude annual mortality rate in a developing country
C. Cause specific death rate as compared to a developed country is mostly due to one of
D. Proportionate mortality the following reasons
A. An incorrect record keeping
Key: B B. A younger age distribution
C. An inaccurate census of the population.
87. The annual report of POF Hospital for the year 2006 shows D. More stressful life style
200 cases of Myocardial Infarction, 35 cases of
Cholecystitis, 105 cases of Pneumonia and 350 cases of Key: B
Acute Gastroenteritis. The result of this report cannot be
generalized on the total population of Wah on account o 93. Birth rates of a population of infants at 40 weeks gestational
A. Confounding bias age are approximately normally distributed, with a mean of
B. Memory bias 3000 grams. Roughly 68% of such infants weigh between
C. Selection bias 2500 and 3500 at birth. If a sample of 100 infants was
D. Berkesonian bias studied, the standard error would be
A. 50
Key: D B. 100
C. 250
88. Increased number of Malaria cases was reported in the time D. 500
intervals between Augusts to October and March to April.
There are 2 different periods in a year, where increased Key: A
malaria transmission is reported. Such an occurrence of
malaria cases in time will give a distribution which is 94. The estimate of the average number of additional years a
A. Unimodal person could expect to live if the age specific death rates for
B. Sporadic a given year prevail for the rest of his life, is best expressed
C. Bimodal by
D. Endemic A. Survival index
B. Probability of dying
Key: c C. Life expectancy
D. Crude death rate
89. In the medical OPD of teaching hospital of Wah Medical
College, Wah Cantt, diastolic blood pressures of 10 patients Key: C
were as follows: 80, 75, 81, 79, 71, 95, 75, 77, 84 & 90. The
mean of this data is 95. A patient who has come from India, reports to a health
A. 80 facility with generalized aches and pain and rash on the
B. 81 body excluding palms and soles. The most likely diagnosis
C. 82 is
D. 83 A. Scarlet fever
B. Trypanosomniasis
Key: B C. Malaria
D. Dengue
90. In a poor community, there is high prevalence of acute
diarrhea cases. The best method for preventing this health Key: D
problem in the long run is
A. Anti-diarrheal drugs. 96. In a medical journal report, the observed mortality of
B. Immunization against cholera and typhoid smokers and nonsmokers for laryngeal squamous cell
C. Provision of sanitary latrine carcinoma was reported to be significant at p < 0.05. Such a
D. Use of boiled water. statement means that
A. The investigator is rejecting the null hypothesis even
Key: C though the results could have occurred purely by
chance a maximum of 5 times out of 100.
91. In a sample of 49 individuals, the mean total leukocyte B. There is a difference between the mortality rates of
count is found to be 7600 cells /mm3, If total leukocyte smokers and nonsmokers 5% of the time
count follows a normal distribution curve, the 50 % of the C. The null hypothesis claims that there is a difference
individuals will have which of the following values: between the mortality rates of smokers and
A. Between 6200 and 9000 nonsmokers.

377
D. A causal relationship between smoking and mortality
may be established through this study 102. If, in one of the groups of premature infants, the maximum
value for hexosaminidase-A was substituted with a much
Key: A higher value, which of the given values remains unchanged
A. Variance
97. In study carried out in the hospital ward, every 10th B. Range
admitted patient was included in the sample, which C. Standard deviation
sampling procedure is this D. Median
A. Random sampling
B. Stratified sampling Key: D
C. Quota sampling
D. Systematic sampling 103. The smoking history of pregnant women is taken in the
antenatal period and correlated with the birth weight at the
Key: D time of delivery. To find an association between them
would be an example of
98. In a sample of 49 individuals, the mean total leukocyte A. Clinical trial
count is found to be 7600 cells /mm3 and standard deviation B. Nested cohort study
of 1400 /mm3, a randomly selected individual will have C. Retrospective study
total leukocyte count lower than 4800 cell /mm3 D. Prospective study
A. 1 % of the Time
B. 2.5 % of the Time Key: D
C. 5 % of the Time
D. 10 % of the Time 104. In a bulk of hundred children out of whom 28 are
immunized 2 of them get measles simultaneously.
Key: B Subsequently14 get measles. Assuming the efficacy of the
vaccine to be 100%, what is the secondary attack rate
99. In a cohort study concerning the relationship between the A. 5%
use of exogenous estrogens and the subsequent risk of B. 10%
breast cancer, a sample of 1000 premenopausal women C. 20%
were followed for 8 years. The results are presented in the D. 21.5%
table above. The absolute risk of breast cancer among
women who did not receive estrogen therapy is Key: D
A. 0.05
B. 0.2 105. A village has a total of 100 under-five children. The
C. 0.4 coverage with measles vaccine in this age group is 60%
D. 0.6 (assuming the efficacy of vaccine to be 100%). Following
the occurrence of a measles case in a child after a visit
Key: B outside, twenty- six children developed measles. The
secondary attack rate of measles is
100. In a cohort study concerning the relationship between the A. 40%
use of exogenous estrogens and the subsequent risk of B. 56%
breast cancer, a sample of 1000 premenopausal women C. 65%
were followed for 8 years. The results are presented in the D. 66%
table above. The relative risk associated with estrogen
therapy in this study is Key: D
A. 0.33
B. 0.5 106. In an epidemiological study the incidence of disease in
C. 2 females is more than that of males but the prevalence is
D. 3 equal in both. It indicates
A. Case fatality is more in female
Key: D B. Mortality in male is higher
C. Disease is of less duration in males
101. Following a large group of cigarette smokers for a period of D. Females harbour disease for longer duration
10 years to determine the occurrence of chronic obstructive
pulmonary disease (COP, coronary heart disease, and Key: A
various forms of lung cancer would be an example of
A. Randomized clinical trial 107. A total of 300 newly diagnosed patients with laryngeal
B. Cross-sectional study cancer are allocated to treatment with either surgical
C. Prevalence study excision alone or surgical excision plus radiation treatment.
D. Cohort study What is the study design
A. Case series report
Key: D B. Case-control study

378
C. Clinical trial A. Cross sectional
D. Cohort study B. Case report
C. Case control
Key: C D. Incidence

108. An analysis of the race of patients who visit an emergency Key: D


room reveals that 40% are white, 25% are black, 20% are
Native American, and 15% are Asian. These data would 114. Gold standard for breast cancer
best be depicted graphically with a A. Mammography
A. Cumulative frequency graph B. X-ray
B. Normal curve C. USG
C. Histogram D. CT
D. Pie chart
Key: A
Key: D
115. If a researcher wants to study precedence relationship
109. A study was conducted in America to find out the between the exposure and effect, which design should he
proportion of blacks and white Americans in California. prefer
This variable chosen is A. Descriptive survey
A. Nominal B. Ecological survey
B. Ordinal C. Cross sectional
C. Discreet numerical D. Cohort
D. Dichotomous
Key: D
Key: D
116. Smokers have risk of lung cancer four times more than non-
110. A public health physician wants to study the load of smokers. If smoking indicates causal association this
hypertension in Rawalpindi district to establish special characteristic gives
screening & treatment services in the mentioned are which A. Specificity
design is more useful for this B. Strength of association
A. Cross sectional C. Coherence
B. Case series D. Consistency
C. Cohort
D. Case control Key: B

Key: A 117. The incidence of gonorrhoea is continuously increasing in a


particular locality. An Investigator reveals that mostly sex
111. Japan has a high rate of stomach carcinoma and a low rate workers are living there. This epidemic may be classified as
of colon carcinoma than the U.S.Which study would you A. Common source single exposure
suggest to prove or support the environmental effect on the B. Common source continuous exposure
incidence of these cancers C. Propagated epidemic
A. Migrant studies D. Slow epidemic
B. Case control
C. Incidence Key: B
D. Case reports
118. Most important reason for recommending oral polio vaccine
Key: A in the polio eradication campaign despite availability of a
safe injectable vaccine that, is
112. To give the relevant importance to hypertension control in a A. Provides 90% immunity in one dose
health service a researcher wants to study the prevalence of B. Does not interfere with vertical immunity
hypertension. He chose a cohort study. The design to assess C. Has been donated by WHO
prevalence is D. Provides herd immunity
A. Inappropriate
B. Suitable Key: D
C. Quick
D. Feasible 119. A 5 year old child comes to the immunization centre
without BCG scar on his arm; what would you prefer
Key: A A. Give BCG vaccine
B. Perform mantoux if positive then give BCG
113. A researcher wants to study natural history of silicosis in a C. Chemoprophylaxis
population of industrial workers. Which design is most D. Perform mantoux if negative then given BCG
useful

379
Key: D patient. He had completed his course of immunization
against hepatitis B last year. What would you recommend
120. A woman reports for vaccination against tetanus only 25 for him
days before delivery; she has not received the first dose A. Booster dose of HB Vaccine
what will you do— Give one dose of tetanus toxoid and B. Single dose of passive immunization
advice the second dose after delivery C. Both active and passive immunization
A. Give anti-tetanus immunoglobulin D. Two doses of immunoglobulins 30 days a part
B. Give two doses of tetanus toxoid with 2 weeks interval
C. Advise appropriate antibiotic course during delivery Key: D
D. Give one dose of tetanus toxoid and advice the second
dose after delivery 126. A forty years old guard of forestry presented in emergency
with complaint of dog bite on his left leg. On examination a
Key: D deep transdermal wound was found. He gave history of
completing the course of immunization against rabies last
121. A woman in the seventh month of pregnancy reports to you year. His serum antibody titre was > 0.5 IU/ml of blood.
in the antenatal clinic for the first time. The recommended What would you advise
immunization is by A. Complete course of active immunization & passive
A. Tetanus toxoid immunization
B. Hepatitis B vaccine B. Only passive immunization
C. Rubella vaccine C. Two booster doses of HDC vaccine
D. Pneumococcal vaccine D. Three booster doses of HDC vaccine + RIG

Key: A Key: C

122. A doctor was attending a patient suffering from Hepatitis B; 127. A 30 years old man went in Benazir Bhutto’s rally at
he accidentally got a prick from a contaminated syringe for Rawalpindi, where in a suicidal attack he got a penetrating
maximum preventive use of Hepatitis B Immunoglobulin injury on his leg. The emergency treatment was given. His
(HBIG), it is given as immunity status against tetanus is not known. The required
A. 0.5 ml / kg body weight within 2 hours anti-tetanus measures are
B. 0.5 ml / kg body weight within 24 hours A. Toxoid one dose
C. 0.06 ml / kg body weight within 3 days of exposure & B. Toxoid one dose + TIG
repeating after one month C. Toxoid complete course
D. 0.06 ml / kg body weight preferably within 3 days D. Toxoid complete course + TIG

Key: C Key: D

123. A mother brought her six weeks old child to an EPI centre 128. A 20 years old lady read an article in a newspaper on
for routine immunization. She was enquired about history of vaccination against tetanus. She is very conscious of herself
Epilepsy in the family and febrile fits. The doctor took this being vaccinate The best schedule that you would suggest
history to avoid complication with for her at this age is:
A. Diphtheria toxoid A. Single dose of TT
B. Tetanus toxoid B. Two doses of TT one month apart
C. Hepatitis B vaccine C. Two doses one month apart with booster after five
D. Pertussis vaccine years
D. Five doses of TT
Key: D
Key: D
124. A conference is being held for prevention of haemorrhagic
fever in our country. The people from South Africa are also 129. A primigravida came for antenatal. Her base line
invite your opinion as a health expert is sought regarding investigations along with screening for Hepatitis B and C
transmission of prevalent infection in South Africa to were done. She was diagnosed HBV positive. What
Pakistan. The infection that you should be most concerned measure would you suggest to prevent the infection in her
about is—Yellow fever child after delivery
A. Dengue fever A. Active immunization only
B. Typhus B. Active & passive Immunization
C. Hay fever C. Only passive immunization
D. Yellow fever D. Chemoprophylaxis

Key: D Key: B

125. An M.S student of Wah cantt Medical College got an 130. A 6 weeks old boy came for DPT, polio & HBV vaccination.
accidental prick while drawing blood of hepatitis-B positive He was given initial doses of all and was called after 4

380
weeks to have the next doses. The likely reason for calling
him again was 136. In Sir-Syed Model School a student of class 3 developed
A. Loss of immune memory measles. The child was isolated from rest of the class. The
B. Stimulation of macrophages school medical officer advised for the contest of this child—
C. Summation of immune responses Passive immunization
D. Replication of lymphocytes A. Passive immunization
B. Active immunization
Key: C C. Anti-viral therapy
D. Advice for active and passive immunization of the
131. A mother brought her four year old child to the doctor. She baby at birth.
gave the history that her child was in close contact with a
case of diphtheria in school. She was very anxious about her Key: D
child and gave history of booster dose of DT 2 years ago.
What would be line of management for such a child 137. In Army public School a student of class 3 developed
A. Booster dose of DT with penicillin measles. The child was isolated from rest of the class. The
B. Active and passive immunization school medical officer advised for the contest of this child:
C. Active and passive immunization with A. Active immunization within 3 days
chemoprophylaxis B. Passive immunization
D. Only keep under surveillance for 1 week C. Chemoprophylaxis
D. Isolation
Key: A
Key: B
132. A General Practitioner purchased BCG vaccine for his
clinic He should store this vaccine at his clinic in 138. A primigravida delivered a baby boy in the obstetric ward
A. A dark place of POF hospital the doctor referred the baby on the same
B. Deep freezer day to the EPI centre for vaccination of
C. Water carrier A. OPV only
D. Refrigerator B. BCG OPV
C. DPT only
Key: D D. DPT HBV +OPV

133. A 5 years old boy is brought to the emergency department Key: B


with history of dog bit Examination revealed multiple
transdermal bites on left leg. The best management for such 139. The population living in Wah Cantt is using water from a
a patient is deep spring, which is considered to be relatively free from
A. Anti-rabies serum, suturing of wound, TT organic contamination but rich in calcium bicarbonates and
B. Anti-rabies serum, suturing of wound, Vaccine sulphates. On account of properties that spring water has,
C. Vaccine, leave wound open, TT, ARS protects people from
D. Vaccine, leave wound open, TT A. Gastroenteritis
B. Ancylostomiasis
Key: C C. Atherosclerosis
D. Renal problems
134. 1 year old child is being treated in Shaukat Khanam
Hospital Lahore and getting radiotherapy for carcinoma. A Key: C
polio case has been detected in his residential locality.
Pediatrician decides to protect him against poliomyelitis by 140. A well was present in a rural area where an unsanitary bore-
giving hole latrine with lots of flies was present within 10 feet of
A. Human normal immunoglobulin distance. The disease more likely to be transmitted through
B. Human specific immunoglobulin drinking this well water is
C. Oral polio vaccine A. Leishmaniasis
D. Inactivated polio vaccine B. Typhoid
C. Dental caries
Key: D D. Ancylostomiasis

135. A 6 months pregnant lady comes for antenatal checkup for Key: B
the first time. Her baseline investigation and screening for
HBSAG and Anti HCV was done. On screening she was 141. If a child has been drinking water containing 30 mg/L of
found to have HBSAG. What would you prefer for this nitrates the condition likely to occur Is
lady— Advice for active and passive immunization of the A. Infantile Methemoglobinemia
baby at birth. B. E-coli enteritis
C. Botulism
Key: D. Dental caries

381
Key: A
Key: A
147. A dental surgeon appointed in rural health centre reports an
142. Required amount of chlorine was added to a large body of increased incidence of dental carries in the children of that
water after sedimentation. The pH of water was 4.0 and are the relevant preventive measure that he should suggest
level of sulphides was negligible. A contact period of one to the health authorities is
hour was ensure. Eventually, it was found that chlorination A. Fluoridation of water
was not successful. The likely reason was B. Chlorination of water
A. Low pH C. Use of bacterial filter
B. Less contact time D. Use of boiled water
C. Less amount of chlorine
D. Suspended impurities Key: A

Key: A 148. Chlorination of water was done by addition of bleaching


powder solution containing 10% available chlorine One
143. People of a village reported a high prevalence of bacterial hour contact time was ensure What is your recommendation
gastroenteritis even after proper chlorination of water regarding use of this water for drinking
supply for the recommended duration. On water analysis, A. Fit for consumption
level of chlorine in water was 0.01 mg/L and pH of water B. Use after 06 hours
was 6.5. There were no suspended impurities; levels of C. Use after 12 hours
sulphides and ferrous were low. The likely reason of D. Rechlorinate
increased bacterial gastroenteritis even after chlorination is
A. Low residual chlorine Key: D
B. High pH leading to chlorination failure
C. Presence of sulphides 149. Water samples from two villages of Punjab were sent to
D. Low level of ferrous Health laboratory for examination. Lab reports show
fluoride levels ranging from 5.26 to 6.32 mg/lit. Use of this
Key: A water for drinking may lead to
A. Dental caries
144. A water sample was taken from a source where catchment B. Dental fluorosis
area included a large agricultural land. It was declared unfit C. Gingivitis
for human consumption on account of raised concentration D. Periodontitis
of a chemical. The likely chemical which has resulted in
making this water unfit is Key: B
A. Iodine
B. Calcium 150. An outbreak of scabies was reported in a Kachi abadi
C. Zinc consisting of 500 people. The appropriate preventive
D. Nitrite measures suggested by you would be to
A. Filter the water
Key: D B. Improve accessibility to water
C. Destroy breeding sites of insects
145. During a sanitary inspection of a rapid sand filtration plant, D. Chlorinate water
slowing of the filtration rate was observed owing to loss of
head. Which method will you suggest to give head to water Key: B
in such a situation
A. Addition of alum 151. A sample of water taken from a water storage tank of a
B. Scraping the top layer residential area was to be examined bacteriologically. A
C. Increasing duration of storage positive test of water sample by multiple tube method refers
D. Back washing of sand bed to the presence of
A. Coli-form organisms
Key: D B. Fecal streptococci
C. Nitrites
146. You were required to chlorinate well water; you added D. Cl. Perfringens
required amount of bleaching powder solution to the water
and allowed an overnight contact time. What is your Key: A
recommendation regarding consumption of this water for
drinking 152. An epidemic of gastroenteritis affected more than 500
A. Fit for consumption people in a city. Samples of water were taken from different
B. To be used after 12 hours sites of the supply system. Bacteriological examination was
C. To be used after another 24 hours positive for coli forms. Chemical analysis of water showed
D. Rechlorinate the presence of high fluoride, nitrate, chloride and ph of 6.
Which content is suggestive of water born epidemic

382
A. Nitrates A. Psychosocial
B. Chlorides B. Malnutrition
C. High pH C. Typhoid
D. Coli-forms D. Enterobius vermicularis

Key: D Key: A

153. Different agents are used for chlorination of water on large 159. A water sample was taken from a village near Taxila On
scale. If after chlorination taste of water is not much altered; chemical analysis the fluoride level was found to be 0.03
level of residual chlorine is more stable and persistent. The mg/lit. The likely effect on the body is
likely agent to be used for chlorination was powder A. Dental flourosis
A. Bleaching powder B. Dental caries
B. Chlorine gas C. Skeletal flourosis
C. Chloramines D. Caries spine
D. Perchloron
Key: B
Key: A
160. An army troop while being transported to Himalayan station
154. In a poor community, there is high prevalence of acute had to stay at the altitude of12,000 feet for 04 days. One of
diarrhea cases. The best method for preventing this health the soldiers developed pulmonary edema. The best
problem in the long run is immediate measure to be taken is
A. Anti-diarrheal drugs. A. Antibiotic therapy
B. Immunization against cholera and typhoid B. Suction of pulmonary fluids
C. Provision of sanitary latrine C. Artificial respiration
D. Use of boiled water. D. Shift the patient to lower altitude

Key: C Key:

155. Six of the ten family members living in a single room house 161. A Person working in compressed air chamber presented
complain of intense itching with scratching in axillae, groin with symptoms of cough, dyspnoea and joint pains. This
and hands; it is more marked at night. The most likely clinical presentation is suggestive of
diagnosis is A. Pulmonary edema
A. Scabies B. Fat embolism
B. Dermatitis C. Air embolism
C. Eczema D. Rupture of spleen
D. Psoriasis
Key: C
Key: A
162. The atmospheric pressure at earth’s surface close to the sea
156. In a house consisting of two living rooms, the door and level averages 760 mm of Hg. If a man lives at an altitude
windows are facing each other. This will provide of 13000 feet above the sea level for few years, the main
A. Low humidity physiological effect is—Increase in concentration of
B. Aspiration hemoglobin
C. Cross ventilation A. Decrease in respiration
D. Diffusion B. Increase in concentration of hemoglobin
C. Decrease in concentration of hemoglobin
Key: C D. Decrease in cardiac output

157. A 12 member’s family was living in a house consisting of Key:


two rooms. Which disease is most likely to be common in
the given situation 163. Husband and wife belonging to low socioeconomic status of
A. Asthma a village are brought to the hospital with mental confusion,
B. Tuberculosis loss of memory, labored breathing eventually leading to
C. CA Bronchus coma. There is history of using coal fire, what is the most
D. Cystic fibrosis probable diagnosis
A. CO2 poisoning
Key: B B. CO poisoning
C. Hydrogen sulphides poisoning
158. A 5 member family was residing in a small house. The D. Sulphur dioxide poisoning
available floor space to one person was 30 sq ft. The
problem which is more likely to be associated with this Key: B
available space is

383
164. A survey on air pollution was conducted in an urban area to understand the use of ORS. What method can provide the
determine the major contaminant in air. The major best solution in this scenario
contaminant estimated in all air pollution is A. Role playing
A. Grit and dust B. Poster competition
B. Sulphur dioxide C. Radio Programme
C. Carbon monoxide D. Lectures
D. Nitrogen dioxide
Key: A
Key: B
171. There was a tableau held in CMH Hospital, Wah Cantt on
165. A person presented with hearing loss after 12 hours of noise the world children’s day to demonstrate the importance of
exposure. This hearing loss disappeared shortly afterwards. ORS in dehydration with a back drop of ORS, breast
The frequency of noise is likely to be between the ranges of feeding and MCH service. This method is known as
A. 1000 – 2000 Hz A. Poster competition
B. 2000 – 4000 Hz B. Role playing
C. 4000 – 6000 Hz C. Symposium
D. 6000 – 8000 Hz D. Lectures

Key: C Key: B

166. To impart health education regarding child care to large 172. After a thorough study of socio-demographic characteristics
number of mothers visiting MCH centre it is decided to of a population in Dhok Ratta, a relevant method of health
resort to method of group discussions. What could be education against smoking was employed to this population.
appropriate strength of each group for this purpose Upon assessing the population habits even after lapse of 2
A. 3-5 years, no change in the behaviour of the smokers was note
B. 4-6 what is likely to be missing in this programme to achieve
C. 6-12 the desired results
D. 20-25 A. Reinforcement
B. Knowledge of cultures
Key: C C. Knowledge of beliefs
D. Required devotion
167. To develop effective AIDS control strategy for Pakistan, the
experts from all over the world are invited to devise the plan Key: A
after relevant discussion. What name is given to this method
A. Seminar 173. In Pakistan about 50% population is illiterate and smoking
B. Group discussion is mostly prevalent among the poor. Government has started
C. Symposium giving pictorial warnings instead of written, to have a
D. Panel discussion greater impact of health education on people: This strategy
is more focused on
Key: D A. Message
B. Receiver
168. Persuasive communication was deliberately employed to C. Sender
manipulate feelings, attitudes and beliefs of people D. Channel
regarding smoking, this method is known as
A. Counseling Key: B
B. Motivation
C. Propaganda 174. A pregnant woman presented with pallor, shortness of
D. Advising breath, lethargy and palpitation. Her blood examination
revealed hemoglobin level of 9 gm/dl. In your opinion what
Key: C could be the probable deficiency in the woman—Iron
deficiency
169. Health education is the responsibility of A. Iron
A. Health educationalist B. Niacin
B. Doctor C. Vitamin C
C. Paramedical staff D. Iodine
D. Every health worker
Key: A
Key: D
175. A 30 years old pregnant lady, gravidity 2 parity 1, visited
170. Diarrhoeal cases among children of an urban slum are on a the antenatal clinic at 32 weeks of gestation. She was a
rise. Almost all the mothers are illiterate and belong to known smoker. On physical examination blood pressure
lower socioeconomic class. It seems difficult to make them was found to be 120/80 mm Hg. Blood examination showed

384
Hb level of 12.5 g/dl. Ultrasound was suggestive of depression. This contraceptive method should be used
intrauterine growth retardation. Which factor during cautiously in this woman because of her— Age
pregnancy would have contributed to intrauterine growth A. Age
retardation—Smoking B. Excess weight
A. Parity C. Low weight
B. Anemia D. Beetle chewing habit
C. Hypertension
D. Smoking Key: A

Key: D 181. For long term contraception, a sub -dermal implant known
as “nor-plant” is being used now a days. The main
176. A 28 years old pregnant lady, gravidity 1 parity1, visited the disadvantage of Norplant is
antenatal clinic at 32 weeks of gestation. She was a known A. Spontaneous expulsion
alcoholic. On physical examination blood pressure was B. Irregular menstrual bleeding
found to be 120/80 mm Hg. Blood examination showed Hb C. Pelvic inflammatory disease
level of 12.5 g/dl. Ultrasound was suggestive of intrauterine D. Ectopic pregnancy
growth retardation, Microcephaly and developmental delay.
Which factor during pregnancy would have contributed to Key: B
the fetal conditions
A. Maternal age 182. A primary Gravida with O-ve blood group comes to labour
B. Parity room with labour pains. Her husband’s blood group is O+v
C. Anemia In order to avoid Rh incompatibility in pregnancy, which
D. Alcohol consumption measure would you take
A. Administer AntiRh immunoglobulins at delivery
Key: D B. Serial ultrasonography
C. Chorionic villous biopsy
177. A newly married couple visits family planning centre for D. Amniocentesis
contraception. Upon investigation the woman has
hyperlipidemia. The method of contraception which would Key: A
be used cautiously in this woman is
A. Condom 183. An 8 weeks pregnant lady comes to gynae OPD for
B. Combined oral contraceptive pill antenatal visit for the first time. She told the doctor that she
C. Second generation IUCD has come from far off area and it’s not possible to come for
D. Mini pill her monthly antenatal visits. The doctor advised the
minimum number of essential antenatal visits is
Key: B A. 3
B. 4
178. A 35 years old woman was brought to hospital with toxic C. 5
shock syndrome. She was using some method of D. 6
contraception. The method of contraception which had
likely lead to this condition is Key: A
A. Condom
B. Diaphragm 184. A 25 years old para 2 delivered a baby boy last month. She
C. Vaginal douching went to a family planning center to seek advice for
D. Oral pills contraception. The frequency of breast feeding at night is
more. The best way to avoid conception in the first four
Key: B months following delivery is the use of
A. Oral contraceptive
179. A 30 years old lady having two kids wants to plan her B. Injectable contraceptive
family. On examination she is found to be anemia. She also C. 3rd generation IUCD
gives history of ectopic pregnancy last year. The best D. no contraception
method of contraception for her would be
A. Lippe’s loop Key: D
B. Injectable contraceptives
C. Progestasert 185. A nulliparous woman is presented in the OPD with severe
D. Multi-load bleeding after taking some contraceptive method. The
gynecologist mimetically assessed that cause of bleeding
Key: B was
A. Copper T
180. A 42 years old female wants to use oral contraceptive pills. B. Oral contraceptive
She is 55 kg of weight and beetle chewer. On general C. Lippe’s loop
examination she was found to be suffering from mild D. Norplant

385
of appetite and abdominal colic of 2 weeks duration. You
Key: C will prefer to investigate him for
A. Cholecystitis
186. A primigravida with 30 weeks of gestation visit OPD for B. Lead poisoning
routine checkup. Her P was found to be 160/90 mm Hg. The C. Appendicitis
doctor advised to report immediately if she develops D. Ameobiasis
A. Abdominal pain
B. Vaginal bleeding Key: B
C. Tinnitus
D. Swelling of the feet 192. In a lead pipe factory, you want to carry out a screening
programme in workers to exclude lead poisoning. Your
Key: D choice of the most useful screening test will be
measurement of
187. A person aged 40 years, working as a laborer in grain A. Lead in blood
market for the last 25 years presented with a history of B. Lead in urine
repeated attacks of respiratory infections in the last 1 year. C. Coproporphyrin in urine
X-ray showed pulmonary fibrosis. The likely diagnosis was D. Basophilic stippling of RBCs
A. Tuberculosis
B. Silicosis Key: C
C. Silicotuberculosis
D. Farmers lung 193. In an automobile manufacturing plant of Pakistan, a large
number of employees are working in different sections.
Key: A Persons who work in the section of welding the parts
together have started reporting sick with redness of eyes. On
188. An industrial worker reported to you with complaints of examination conjunctivitis and keratitis are found. What
cough, history of dyspnoea on exertion and pain in the chest. could be the cause of this problem
His X-ray chest showed snow storm appearance. The A. Poor Personal Hygiene
diagnosis would be— Silicosis B. Heat
A. Asbestosis C. Overwork
B. Siderosis D. Ultraviolet radiations
C. Silicosis
D. Byssinossis Key: D

Key: C 194. Screening is done


A. To detect disease at its early stage
189. An occupational worker presented with complaints of B. To detect disease in late stages
exertional dyspnoe He gave history of being in an industry C. To detect the outcome of disease
dealing with spare parts such as gas kit and brakes, he also D. None
gave history of smoking for about five years. His X-ray
chest showed a ground glass appearance / honey combing in Key: A
the lower two thirds of the lung fields. The likely condition
that he suffers from is—Asbestosis 195. Study in which every person of a population has equal
A. Silicosis chances of being selected
B. Anthracosis A. Random sampling
C. Asbestosis B. Stratified sapling
D. Siderosis C. Systemic sampling
D. None
Key: C
Key: A
190. The health of mothers are improved and Maternal and child
health care is one of the main components of (PHC) systems 196. Your opinion is sought as a Public Health Specialist by the
as declared at the Alma Ata Conference 1978 in employers of a glass factory in which some workers have
A. USA developed Silicosis. The most important control measure
B. United kingdom that you recommend is
C. Kazakistan A. Pre-placement examination
D. China B. Adequate personal hygiene
C. Periodic X-ray chest
Key: C D. Rigorous dust control

191. A worker who had been in the battery manufacturing unit Key: D
for the last 20 years, reported to you with complaints of loss

386
197. 20 workers of a chemical factory located in an industrial 203. A pregnant lady reported to ante natal clinic with signs and
area near Lahore, are handling irritant chemicals symptoms of Rubella during 28th week of gestation. She was
(dichromates) as part of their to such workers require insisting upon the termination of pregnancy as she feared
periodic medical examination. What could be the congenital malformations of fetus. Doctor told her that
appropriate frequency of such examinations in your opinion maximum time for Rubella to cause major abnormalities of
A. Once a year the fetus during pregnancy is till:
B. Twice a year A. 8th week
C. Monthly B. 12th week
D. Weekly C. 16th week
D. 20th week
Key: D
KEY: C
198. The commonest physical health hazard in most industries is
A. Heat 204. In a poor community, there is high prevalence of acute
B. Noise diarrhea. The best method for preventing this health problem
C. Humidity in the long run is:
D. Ionizing radiation A. Anti-diarrheal drugs.
B. Immunization against cholera and typhoid
Key: A C. Provision of sanitary latrin
D. Use of boiled water.
199. A worker of a brick kiln was brought to the emergency
department, in an unconscious stat He was hypotensive and KEY: C
sweating profusely. The likely condition he suffered from
was 205. A woman of 70 kg reports to a health facility at term. On
A. Heat stroke examination her P was 130/90 mm of Hg; fundal height was
B. Heat exhaustion more than the gestational age. A large single baby was seen on
C. Heat Hyperpyrexia ultrasound with no other fetal abnormality. The likely
D. Heat cramps condition is: ( NUMS PMDC 2019)
A. Maternal diabetes
Key: B B. Hypertension
C. Obese mother
200. A patient is suffering from AIDS. He is suffering from D. Hydramnios
suppression of which of the following cells?
A. Monocytes KEY: A
B. T- Cells.
C. B-Cells 206. A woman of 40 in the seventh month of pregnancy reports
D. CD4 Cells to you in the antenatal clinic for the first time. The
recommended immunization is by:
KEY: D A. Tetanus Toxoid
B. Hepatitis B vaccine
201. 15 years old boy, resident of Murree is having swelling in C. Rubella vaccine
the neck. Apart from weakness and D. Pneumococcal vaccine
constipation; there are no other complaints at present. He is
likely to be suffering from deficiency of: (NUMS PMDC KEY: A
2019)
A. Calcium 207. A 30 years old woman fitted with an IUCD comes for post
B. Fluorine insertional examination and complains of vaginal bleeding;
C. Iodine which of the following is the least important to look for in
D. Iron this patient:
A. Break through vaginal bleeding
KEY: C B. Pelvic infection
C. Dysmenorrhea
202. 50 years old Bank officer was told to be suffering from D. Endometrial cancer
illeocecal Tuberculosis. Nobody suffered tuberculosis in the
family, but he still got infected, probably due to: KEY: D
A. Infected milk
B. Infected clients 208. One year old child comes to emergency with history of
C. Contaminated vessel watery diarrhea and high grade fever for the last 24 hours. A
D. Stressful work house physician observes his impalpable pulse and un-
recordable blood pressure The first immediate step in
KEY: A emergency is to advise: (NUMS PMDC 2019)
A. I/V Ringers lactate

387
B. Oral rehydration therapy KEY: A
C. I/V antibiotics
D. Stool examination 215. The intermediate host involved in transmission of
Diphyllobothriasis
KEY: A A. Snail
B. Swine
C. Fish
209. A mother brought her six months old child to a BHU. She is D. Dog
worried about the growth of her
child. The best single measure for assessing the physical KEY: C
growth in this age is: (NUMS PMDC 2018)
A. Weight for age 216. Koplik's spots are the diagnostic sign of one of the following
B. Height for age diseases: (NUMS PMDC 2019)
C. Head chest ratio A. Rubella
D. Weight for height B. Whooping cough
C. Measles
KEY: A D. Mumps

210. A woman reports at a health facility with labour pains at term. KEY: C
She is suffering from chicken pox. The measure taken to
prevent chicken pox in the baby after birth is to give: 217. The best measure taken in meningococcal meningitis out-
A. Acyclovir to woman breaks is:
B. HZIG to baby at birth A. Mass chemo-prophylaxis by penicillin
C. HZIG to woman before delivery B. Immunization
D. HZIG and acyclovir to mother C. Chlorination of all water supplies
D. Mass screening program
KEY: B
KEY: B
211. The most common cause of early neonatal mortality is:
A. Low Birth Weight 218. The best strategy for the control of pulmonary Tuberculosis
B. Acute Respiratory Infections as recommended by WHO is:
C. Malnutrition A. BCG vaccination at mass level
D. Jaundice B. Chemoprophylaxis with INH
C. Early case detection and prompt treatment
KEY: A D. Proper disposal of sputum

212. Erythroblastosis fetalis can be prevented in the next baby, if KEY: C


the mother is injected, at parturition, with an antibody called:
(NUMS PMDC 2017) 219. Bloody diarrhea is seen in one of the following infections:
A. Nonspecific immunoglobulin A. Shigellosis
B. Rho ( immunoglobulin (RhoGAM) B. Botulism
C. Antilymphocyte globulin C. Cholera
D. Antithymocyte serum D. Typhoid

KEY: B KEY: A

213. Which of the following statements about polyvalent anti- 220. Which one of the following indices used in malaria survey
snake venom serum is true? is called Transmission index
A. It is to be given subcutaneously (NUMS PMDC 2019)
B. It is an example of passive immunization A. Infant parasite rate
C. It is usually obtained from human donors B. Parasite rate
D. It gives lifelong immunity C. Spleen rate
D. Sporozoite rate
KEY: B
KEY: A
214. Which of the following is not true for snake bite in
Pakistan? 221. The mode of transmission most difficult to prevent is:
A. Most death occur due to rattle snake A. Person-to-person spread
B. Snake bite is more common in summer B. Droplet spread
C. Management of snake bite requires anti-snake venom C. Vector borne spread
D. Cobra bites are characterized by neurological signs D. Transmission through soil

388
KEY: D
KEY: B
222. Orchitis is the complication commonly associated with
(NUMS PMDC 2019) 229. Diphtheria transmit through s: (NUMS PMDC 2019)
A. Measles A. Droplet
B. Mumps B. Vertical
C. Rubella C. Transplacental
D. Diphtheria D. Contact

KEY: B KEY: A

223. The drug of choice for carriers and contacts in 230. Herpes simplex virus transmitted by
meningococcal meningitis is (NUMS PMDC 2019) A. Vertical
A. Penicillin B. Droplet
B. Chloramphenicol C. Transplacental
C. Rifampicin D. Soil
D. Cephalosporin
KEY: B
KEY: C
231. Folic acid supplementation during lactation period is:
224. “Any loss or abnormality of psychological, physiological or A. 100 mg/d [31]
anatomical structure or function” is known as: (NUMS B. 150 mg/d
PMDC 2019) C. 400 mg/d
A. Injury D. 450 mg/d
B. Impairment
C. Disability KEY: B
D. Handicap
232. Elemental iron supplementation in Iron deficiency anemia is:
KEY: C A. 300 – 400 mg
B. 150 – 200 mg
225. Removal of IUCD is strongly indicated in case of: (NUMS C. 100 – 150 mg
PMDC 2019) D. < 100 mg
A. Uterine bleeding
B. Uterine cramps KEY: C
C. Pregnancy
D. Purulent vaginal discharge 233. Most common cause of low birth weight baby in Pakistan is:
A. Prematurity
KEY: A B. Infection
C. Anemia
226. Recommended dose for iron during pregnancy is: D. Diabetes
A. 24 mg/day
B. 32 mg/day KEY: A
C. 40 mg/day
D. 60 mg/day 234. Minimum ANC visits during pregnancy should be:
A. 3
KEY: B B. 5
C. 9
227. A 45 years old man has weight of 70 kg and height of 1.6 m. D. 12
His BMI is:
A. 24 KEY: A
B. 27.3
C. 30 235. Daily need of calories in pregnancy is:
D. 34 A. 1500 kCals
B. 2000 kCals
KEY: B C. 2500 kCals
D. 3500 kCals
228. A 55 year old female, her weight is 88kg height 172cm. Her
BMI is: (NUMS PMDC 2019) KEY: C
A. 33
B. 30 236. Leading Cause of maternal deaths in Pakistan is:
C. 27 A. Anemia [49]
D. 25 B. Hemorrhage

389
C. Sepsis
D. Obstructed labour 244. When a new treatment is developed that delays death but
does not produce recovery from a chronic disease, which of
KEY: B, PPH, APH the following will occur?
A. Prevalence of the disease will decrease
237. Commonest cause of neonatal mortality in Pakistan is: B. Incidence of the disease will increase
A. Diarrheal diseases C. Prevalence of the disease will increase
B. Birth injuries D. Incidence of the disease will decrease
C. Low birth weight
D. Congenital anomalies Key: C

KEY: C 245. Which one of the following gives the strength of association
between the cause and the disease?
238. Under National Immunization Schedule, total dose of A. Prevalence rate.
Vitamin-A given to a child is: B. Case fatality rate.
A. 5 lac IU C. Adjusted rate.
B. 6 lac IU D. Incidence rate.
C. 9 lac IU
D. 13.5 lac IU Key: D

KEY: C 246. Which one of the following is not included in analytical


epidemiological studies?
239. Earliest feature of vitamin A deficiency is: A. Historical cohort studies
A. Dryness of conjunctiva B. Cohort studies
B. Nyctalopia C. Case control
C. Keratomalacia D. Case report
D. Hyphema
Key: D
KEY: A
247. Killing power of a disease is best expressed by:
240. Most common complication of mumps in children is: A. Attack Rate.
A. Pneumonia B. Secondary Attack rate.
B. Pancreatitis C. Case fatality rate.
C. Meningitis D. Survival Index.
D. Encephalitis
Key: C
KEY: C
248. Difference in incidence rate of disease between exposed
241. Most serious complication of Measles is: group and non-exposed group is:
A. Koplik spots A. Attributable fraction
B. Parotitis B. Relative risk
C. Meningoencephalitis C. Attributable risk
D. Nephritis D. Prevalence rate

KEY: C Key: C

242. Diphtheria carrier are diagnosed by 249. The best strategy recommended by WHO for the control of
A. Throat culture pulmonary Tuberculosis is:
B. Gram’s staining A. BCG vaccination at mass level
C. Albert’s staining B. Chemoprophylaxis with INH
D. Schick test C. Early case detection and prompt treatment
D. Proper disposal of sputum
KEY: A
Key: C
243. The most appropriate test to assess the prevalence of
tuberculosis infection in a community is 250. Which one of the following diseases does not spread by
A. Mass miniature radiography infected droplets?
B. Sputum examination A. Chicken pox
C. Tuberculin test B. Measles
D. Clinical examination C. Poliomyelitis
D. Mumps
KEY: C

390
Key: C
258. A student has come to Pakistan from Kenya, he must have a
251. Which one of the following diseases will not have a human valid certificate of vaccination against:
healthy carrier? A. TB.
A. Cholera B. Pollomyeiitis
B. Diphtheria C. Mumps
C. Salmonellosis D. Yellow fever
D. Hydatidosis
Key: D
Key: d
259. Which of the following data is not measurable?
252. Which one of the following factors will lead to hypertension? A. Nominal
A. High Protein diet B. Ordinal
B. High sodium intake C. Interval
C. Body mass index of 25 D. Continuous
D. Hypervitaminosis A
Key: A
Key: B
260. Which of the following homogeneous data?
253. Which one of the following is not a risk factor for coronary A. Multi modal curve
heart disease? B. Normal distribution curve
A. Oral contraceptives C. Right skewed curve
B. Alcohol abuse D. Left skewed curve
C. High fiber diet
D. Type-A personality Key: B

Key: C 261. Which one of the following is not a measure of central


tendency?
254. The number of Tetanus Toxoid doses for a woman of A. Mean
childbearing age is? B. Median
A. One C. Mean deviation
B. Two D. Geometric mean
C. Three
D. Four Key: C

Key: C 262. Shape of normal distribution curve is based mainly on:


A. Mean and sample size
255. Which one of the following is a cell derived vaccine? B. Mean and standard deviation
A. Measles C. Range and sample size
B. BCG D. Range and standard deviation
C. OPV
D. Hepatitis B Key: B

Key: D 263. The number of deaths in people above 65 years expressed as


percentage of total deaths of all age groups is known as:
256. Which one of the following types of immunization is A. Expectancy of life at 65 years.
available against Rabies? B. Proportional mortality rate.
A. Live attenuated vaccine C. Crude death rate.
B. Killed vaccines D. Specific death rate.
C. Cell derived vaccine
D. Cell wall vaccine Key: B

Key: B 264. Which one of the following is the most reliable indicator of
fertility in a country?
257. Both active and specific passive types of immunization are A. Growth rate
available for which one of the following? B. Crude birth rate
A. Tetanus C. Total fertility rate
B. Meningitis D. General fertility rate
C. Measles
D. Mumps Key: C

Key: A

391
265. The number of daughters a new born girl will bear during 272. Regarding breast feeding which one of the following is true?
her life time assuming fixed age specific fertility and A. Feeding of newborn should commence on the second
mortality rate, refers to which one of the following? day after birth
A. Crude birth rate B. Colostrum contains antibodies
B. Gross reproduction rate C. 5 breast feeds one after every 4 hrs should be given to
C. Net reproduction rate early neonates.
D. Total fertility rate D. Breast feeding is contraindicated in women with
tuberculosis
Key: C
Key: B
266. Vitamin A deficiency state is characterized by one of the
following: 273. Which one of the following result in congenital anomaly?
A. Bitors spots A. Chicken Pox
B. Anemia. B. Candidiasis
C. Diarrhea. C. Typhoid
D. Bone deformities. D. Pneumonia

Key: A Key: A

267. Which one of the following is a meat borne disease?


A. Bacillus Cereus infection 274. Which one of the following is not a mechanical method of
B. Taeniasis. contraception?
C. Ascarlasis. A. Insertion of Copper T
D. Shigellosis. B. Insertion of Lippes Loop
C. Contraceptive implants
Key: B D. Condoms

268. One important clinical sign of Kwashiorkor type of Key: C


malnutrition is:
A. Monkey face 275. A Post coital contraception is recommended within:
B. Oedema A. 06 hours of unprotected- intercourse.
C. Obvious muscle wasting B. 48 hours of unprotected- intercourse.
D. Obvious loss of subcutaneous fat C. 72 hours of unprotected- intercourse.
D. Two days of missed period.
Key: B
Key: B
269. Which one of the following is not a feature of Marasmus?
A. Muscle wasting. 276. Which of the following statements is true for school Health
B. Low weight for height team?
C. Hypo pigmented hair A. It consists of one male doctor & one female doctor and
D. Severe subcutaneous fat loss two nurses.
B. It serves up to 5000 school children.
Key: C C. Every school, small or large, should have its own
school health team linked with BHUs.
270. Which one of the following is not true regarding Vitamin A? D. It is responsible for curative health care.
A. It is a water soluble Vitamin
B. Its deficiency causes xeropthalmia Key: B
C. Animals foods are sources of preformed Vitamin A
(Retinol) 277. Which of the following arrangements regarding desks &
D. Liver stores Vitamin A in the form of retinol palmitate chairs is recommended for school children?
A. Minus type
Key: A B. Plus type
C. Desks with revolving seats
271. The average weight gain for a pregnant woman with BMI D. Desks with recliner chairs
20 - 25 should be:
A. 4 to 8kg Key: A
B. 8 to 12kg
C. 12 to 16kg 278. The Routine Medical Inspection of school children during
D. 16 to 20 kg school career should at least be:
A. Once every year
Key: B B. Four times in the total schooling period
C. On the request of teachers

392
D. In the disease out-breaks. Key: A

Key: B 286. Which one of the following is not an example of organic


dust?
279. The presence of nitrates in water indicates: A. Cane fiber
A. Temporary hardness of water B. Cotton dust
B. Permanent hardness of water C. Grain dust
C. Fresh faecal pollution D. Asbestos
D. Past faecal pollution
Key: D
Key: D
287. Foundry workers may suffer from one of the following
280. Mottling of teeth is a nutrition related condition which diseases as a result of occupational exposure:
results from: A. Silicosis
A. Deficiency of Fluorine in drinking water B. Asbestosis
B. Deficiency of Chlorides in water C. Lung cancer
C. Excess of Fluorine in drinking water D. Anthracosis
D. Excess of Nitrites in water.
Key: A
Key: C
288. Which one of the following is not true regarding
281. The sloughed off layer in the trickling filter during the Pneumoconiosis?
treatment of sewage in modern plants is known as: A. Byssinosis Is due to Inhalation of cotton dust
A. Humus B. Anthracosis is due to inhalation of anthrax spores
B. Wet manure C. Baggassosis is an occupational hazard of workers in
C. Scum paper mills.
D. Sullage D. Asbestosis predisposes to pleural mesothelloma.

Key: A Key: B

282. House waste-water which does not contain human excreta is 289. Which one of the following statements is not true regarding
known as: Lice?
A. Humus A. Lice belong to phylum arthropoda.
B. Sewage B. It undergoes incomplete metamorphosis.
C. Sludge C. Pediculosis is a viral disease transmitted by the bite of
D. Sullage a body louse.
D. Relapsing fever occurs when an infected louse is
Key: D crushed on the skin.

283. Which one of the following does not refer to vitiation of air? Key: C
A. Decrease in air movements
B. Increased concentration of carbon dioxide 290. Which one of the following insecticides is the least toxic
C. Decrease in relative humidity organo-chlorine compound for human beings?
D. Increase in temperature A. DDT
B. HCH
Key: C C. DieIdrin
D. Chlordane
284. Which one of the following is correct regarding daily upper
limit of noise without substantial damage to hearing? Key: A
A. 50 db
B. 75 db 291. Asexual cycle of malarial parasite ends with the production
C. 85 db of which one of the following to propagate the disease?
D. 100 db A. Sporozoties
B. Merozoites
Key: C C. Schizonts
D. Gametocytes
285. Which one of the following is included in pneumoconiosis
a. Anthracosis Key: D
b. Sarcoldosis
c. Psittacosis 292. The best method recommended by WHO for malaria control
d. Aspergillosis in endemic areas is:
A. The use of Pyrethrum impregnated bed nets

393
B. Regular use of Chloroquine C. Coca bush
C. Genetic Control of Mosquitoes D. Mushrooms
D. Vaccination
Key: A
Key: A
299. In a health education programme, if a group of 10 people
293. A snakebite victim was brought to a health facility after 6 are speaking on a topic of common interest, it is called a:
hours of bite. There was a bloody discharge at the site of A. Workshop
bite. There were ecchymosis & serum filled blebs. The most B. Panel discussion
likely snake involved is: C. Group discussion
A. Viper D. Symposium
B. Cobra
C. Kraft Key: C
D. Sea snake
300. In which of the following disasters, injuries usually exceed
Key: A death?
A. Earth quakes
294. Mental retardation is seen in which of the following B. Landslide
childhood conditions? C. Avalanches
A. Trisomy 21 D. Floods
B. Marasmus.
C. Kwashiorkor Key: A
D. Juvenile Diabetes.
301. A 2-year-old female child was brought to a BHU with a
Key: A history of cough and fever for 4 days with inability to drink
for last 12 hours. On examination, the child was having
295. The most important factor which makes the smoking weight of 5 kg and respiratory rate of 45/minute with fever.
cessation program successful is: The child will be classified as suffering from:
A. The desire of the smoker to quit A. Very severe disease
B. A well planned program B. Severe Pneumonia
C. The advice by a competent doctor C. Pneumonia
D. Follow up program D. No Pneumonia

Key: A KEY: A, Very severe Pneumonia - Inability to feed and


severe malnutrition (weight 5 kg at 2 years age) makes the
296. Which one is incorrect about tobacco smoking? child as having Very Severe Pneumonia.
A. The babies born smoking to mothers, who smoke, (NUMS PMDC 2018)
weigh on average 200, less at birth than those of non-
smokers. 302. Restriction of activities of a contact, who has been exposed
B. It is responsible for about 3 million deaths/year in the to a case of communicable disease for maximum incubation
world period, is termed as:
C. In developed world per capita consumption of tobacco A. Elimination
is Increasing B. Quarantine
D. Dose-response relationship is seen In cigarette C. Surveillance
smoking and lung cancer D. Control

Key: C KEY: B

297. Self-administration of a drug for non-medical reasons in 303. The required temperature for sterilization against spores in
quantities and frequencies which may impair an individual's Hot-air oven is:
ability to function effectively and which may result in social, A. 120-140°C for one hour
physical or emotional harm is best named as: B. 140-160°C for one hour
A. Drug dependence C. 160-180°C for one hour
B. Drug tolerance D. 180-200°C for one hour
C. Drug abuse KEY: C
D. Self-medication
304. "Dettol is a commonly used antiseptic agent in the
Key: C community. Its chemical name is:
A. Hexachlorophane
298. Marijuana is derived from which one of the following plants? B. Chloroxylenol
A. Cannabis C. Chlorhexidine
B. Morning Glory D. Cetrimide

394
B. Widal test
KEY: B C. Blood culture
D. Stool culture
305. Our rural masses are facing multiple health problems. The
best approach to deal with their problems is to provide: Key: C
A. Basic health services
B. Health education 312. Which of the following causes toxic shock syndrome?
C. Safe water supply and sanitation A. Klebsiella
D. Primary health care. B. Staphylococcus
C. Streptococcus
KEY: E D. Pseudomonas

306. Which one of the following is not induced in the elements Key: B, Toxic shock syndrome is caused by Staphylococcus
of Primary Health Care? aureus
A. Immunization services
B. Family planning services 313. In sputum culture of a patient of bronchiectasis, most
C. Specialized services common organism seen is:
D. Health education regarding water and sanitation A. Pseudomonas
B. Staphylococcus
KEY: C C. Haemophilus influenza
D. Klebsiella
307. The main headquarters of WHO are present in which one of
the following countries? Key: A, Bronchiectasis Sputum culture is positive for
A. Switzerland Pseudomonas aerugenosa
B. France
C. America 314. Ferruginous bodies are seen in:
D. England A. Silicosis
B. Byssinosis
KEY: A C. Asbestosis
D. Bagassosis
308. Which of the following is diagnostic test for AIDS?
A. Western blot test Key: C
B. CFT
C. Enzyme-linked immunosorbent assay 315. Caplan’s syndrome is seen in:
D. Immunofluorescence assay A. COPD
B. Pneumoconiosis
Key: C, The primary tests for diagnosing HIV and AIDs C. Pulmonary edema
include: ELISA Test — ELISA, which stands for enzyme- D. Bronchial asthma
linked immunosorbent assay, is used to detect HIV infection.
If an ELISA test is positive, the Western blot test is usually Key: B
administered to confirm the diagnosis.
316. All of the following are seen in asbestosis except:
309. Which of the following is the most common causative agent A. Diffuse alveolar damage
of community acquired pneumonia? B. Calcify pleural plaques
A. Staphylococcus C. Diffuse pulmonary interstitial fibrosis
B. Streptococcus D. Mesothelioma
C. Mycoplasma
D. Haemophilus Key: A

Key: B, Most common cause of community acquired 317. Which interstitial lung disease is caused by organic dust?
pneumonia is streptococcus pneumonia (pneumococcus) A. Silicosis
B. Asbestosis
310. Most common cause of osteomyelitis: C. Byssinosis
A. Mycobacterium tuberculosis D. Anthracosis
B. Haemophilus influenza
C. Escherichia coli Key: A
D. Staphylococcus aureus
Cotton dust is associated with
Key: D A. Byssinosis
B. Bagassosis
311. Typhoid fever in first week is diagnosed by: C. Asbestosis
A. Urine culture D. Silicosis

395
A. Prolonged PR interval
Key: A B. Paroxysmal atrial tachycardia
C. Short QT interval
318. Most common presentation of unilateral renal artery D. Bundle branch blocks
stenosis is:
A. Uremia Key: C, Hypocalcaemia typically prolongs the QT interval
B. Hypertension (ST portion), while hypercalcemia shortens it.
C. Peripheral edema
D. Polycythemia 326. Shortest QT interval is seen in:
A. Hypocalcemia
Key: B B. Hypercalcemia
C. Hypokalemia
319. Which of the following is not used in acute asthma? D. Hyperkalemia
A. Ipratoprium bromide
B. Salbutamol Key: B, Hypocalcemia typically prolongs the QT interval
C. Steroids (ST portion), while hypercalcemia shortens it.
D. Montelukast
327. In which of the following, highly “selective” proteinuria is
Key: D seen:
A. Focal sclerosis
320. Pseudofractures are seen in: B. Membranous glomerulonephritis
A. Hyperparathyroidism C. Mesangial prolifertive glomerulonephritis
B. Osteoporosis D. Minimal change disease
C. Osteomalacia
D. Hypoparathyroidism Key: D

Key: C 328. Which of the following are the gram +ve cocci:
A. Pneumococcus
B. Staphylococcus
321. Acute breathlessness is an important feature of: C. Streptococcus
A. Pericarditis D. All of the above
B. CHF
C. Pneumonia Key: D
D. Pulmonary embolism
329. Which of the following is the most common cause of
Key: D bronchiolitis?
A. Influenza virus
322. Most common symptom of pulmonary embolism is: B. Herpes virus
A. Chest pain C. RSV
B. Dyspnea D. Adenovirus
C. Hemoptysis
D. Cough Key: C

Key: B 330. Which of the following is the infective stage of Wuchereria


bancrofti?
323. Most common sign of pulmonary embolism is: A. Microfilaria
A. Sweating B. Third stage larva
B. Tachypnea C. Second stage larva
C. Tachycardia D. First stage larva
D. Cyanosis
Key: B
Key: B
331. Deficiency of Vitamin cause all the following except
324. Best test to detect iron deficiency in community is: A. Decrease synthesis of carnitine
A. Serum ferritin B. Increase collagen synthesis
B. Transferrin C. Biosynthesis of dopamine
C. Hemoglobin D. Decrease collagen synstheis
D. Serum iron
Key: B
Key: A
332. Which of the following mediates cell-mediated immunity?
325. ECG finding in Hypercalcemia is: A. Monocytes

396
B. T-cells
C. B-cells Key: C
D. Polymorphonuclear cells
340. Which of the following is the infective stage of Wuchereria
Key: B, Humeral immunity is mediated by B-cells. Cellular bancrofti?
immunity is mediated by T-cells A. Microfilaria
B. Third stage larva
333. Live vaccine is: C. Second stage larva
A. Salk polio vaccine D. First stage larva
B. Hepatitis B vaccine
C. Yellow fever vaccine Key: B
D. Japanese encephalitis vaccine
341. Deficiency of Vitamin cause all the following except
Key: C A. Decrease synthesis of carnitine
B. Increase collagen synthesis
334. Cholera vaccine is C. Biosynthesis of dopamine
A. Live vaccine D. Decrease collagen synthesis
B. Killed vaccine
C. Bacterial products Key: B
D. All of above
342. Live vaccine is:
Key: B A. Salk polio vaccine
B. Hepatitis B vaccine
335. The first person becoming sick in an epidemic is called C. Yellow fever vaccine
A. Index case D. Japanese encephalitis vaccine
B. Contact case
C. Primary case Key: C
D. Secondary case
343. Cholera vaccine is
Key: C A. Live vaccine
B. Killed vaccine
336. The first case come to the attention of the investigator is: C. Bacterial products
A. Index case D. All of above
B. Contact case
C. Primary case Key: B
D. Secondary case
344. Bence john’s protein in urine can be seen in
Key: A A. Multiple myeloma
B. WaldernstromMacroglobunima
337. In which of the following, highly “selective” proteinuria is C. A and B both
seen: D. None
A. Focal sclerosis
B. Membranous glomerulonephritis Key: C
C. Mesangial prolifertive glomerulonephritis
D. Minimal change disease 345. The first person becoming sick in an epidemic is called
A. Index case
Key: D B. Contact case
C. Primary case
338. Which of the following are the gram +ve cocci: D. Secondary case
A. Pneumococcus
B. Staphylococcus Key: C
C. Streptococcus
D. All of the above 346. The first case come to the attention of the investigator is:
A. Index case
Key: D B. Contact case
C. Primary case
339. Which of the following is the most common cause of D. Secondary case
bronchiolitis?
A. Influenza virus Key: A
B. Herpes virus
C. RSV 347. A postoperative cardiac surgical patient developed sudden
D. Adenovirus hypotension, raised central venous pressure, pulsus

397
paradoxus at the 4th postoperative hour. The most probable C. Varicella
diagnosis is: D. HSV 2
A. Excessive mediastinal bleeding
B. Ventricular dysfunction 355. Hematuria, RBC casts and proteinuria are suggestive of:
C. Congestive cardiac failure A. Nephrotic syndrome
D. Cardiac tamponade B. Acute glomerulonephritis
C. Renal cell carcinoma
Key: D D. Chronic renal failure

348. Best method to diagnose bronchiectasis is: Key: B


A. x-ray
B. Bronchography 356. Japanese encephalitis is caused by
C. MRI A. Human retrovirus
D. HRCT(high resolution CT) B. Enterovirus
C. Flavivirus
Key: D D. Cytomegalovirus

349. Neurofibrillary tangles and plaques are seen in the brain in: Key: B
A. Picks disease
B. Huntington’s chorea 357. Asbestosis predisposes people to
C. Alzheimer’s disease A. Tuberculosis
D. Parkinsonism B. Mesothelioma
C. Lung cancer
Key: C D. Sarcoidosis

350. Onion skin spleen is seen in: Key: B


A. SLE
B. Thalassemia 358. Breast milk as compared to cow’s milk has more of all of
C. ITP the following except
D. Scleroderma A. Calories
B. Fat
Key: A C. Lactose
D. Protein
351. Not seen in Dressler’s syndrome:
A. Usually in 1st or subsequent week after infarction Key: D
B. Ventricular tachycardia
C. Fever 359. One liter of cow’s milk provide….. Mg of calcium
D. Pericarditis A. 400
B. 800
Key: B C. 1000
D. 1200
352. Herpes-zoster most commonly affects:
A. Spinal cord medulla Key: D
B. Dorsal root ganglia
C. Sympathetic ganglia 360. According to WHO, the amount of free chlorine in
D. Anterior horn cells chlorinated water should be?
A. 0.5 mg
Key: B B. 2 mg
C. 3 mg
353. Initial dose of heparin in deep vein thrombosis is: D. 5 mg
A. 10,000 IU/day
B. 1 lakh IU/day Key: A
C. 50,000 IU/day
D. 2 lakh IU/day 361. In clinical setting, the best method of health education is:
A. Counseling
Key: A, Heparin should be administered IV as an initial B. Demonstration
bolus of 7500 to 10,000 IU, followed by a continuous C. Audiovisual
infusion of 1000 to 1500 IU/ hour D. Group discussion

354. Zoster recurrence occurs after infection with: Key: A


A. Small pox
B. HSV 1 362. Cohort study is

398
A. Prospective study
B. Retrospective study Key: A
C. Analytical study
D. All of the above 370. The incubation period of mumps is:
A. 7 days
Key: A B. 10 days
C. 18 days(2 to 3 weeks)
363. The disease which is imported into a country in which it D. 25 days
doesn’t is known as
A. Exotic Key: C
B. Sporadic
C. Epornithic 371. Rabies cannot be transmitted by
D. Enzootic A. Ingestion
B. Aerosol
Key: A C. Licks
D. Bites
364. Which of the following test is done for milk pasteurization:
A. Turbidity test Key: A
B. Phosphate test
C. Nitric acid test 372. Mode of transmission of rabies:
D. Salwotch test A. Animal bites
B. Licks
Key: B C. Aerosol
D. Person to person
365. BCG is administered to children:
A. Intradermal Key:
B. Intranasally
C. Intramuscularly 373. Pellagra is caused by deficiency of:
D. Orally A. Niacin
B. Iron
Key: A C. Calcium
D. Magnesium
366. Montoux test is read after how many hours:
A. 48 Key: A
B. 72
C. 96 374. Which of the following is not involved in the transmission
D. 24 chain of hydatid disease?
A. Swine
Key: B B. Dog
C. Sheep
367. Bagassosis occurs due to inhalation of: D. Cow
A. Cotton dust
B. Sugarcane dust Key: A
C. Silica dust
D. Coal dust 375. Most common site of hydatid cyst is:
A. Brain
Key: B B. Liver
C. Lungs
368. The first scientist to observe bacteria and other microscopic D. Spleen
organism was
A. Antoni van leeuwenhoek Key: B
B. William harvey
C. Virchow 376. Specificity refers to:
D. Syndenham A. True positive
B. True negative
Key: A C. False positive
D. False negative
369. Safe limit of fluorine in drinking water is:
A. 0.5 to 0.8 mg/L Key: B, Specificity is defined as the ability of a test to
B. to 0.3 mg/L identify correctly those patients who do not have the
C. mg/L diseases (true negatives). Sensitivity is defined as the ability
D. 1.5 mg/L

399
of a test to identify correctly all those patients who have the B. Ordinal
disease (true positives). C. Interval
D. Ratio
377. Histogram is used to describe:
A. Quantitative data of a group of patients Key: A
B. Qualitative data of a group of patients
C. Data collected on nominal scale 384. A Scatter diagram is drawn to study:
D. Data collected on ordinal scale A. Trend of a variable over a period of time
B. Frequency of occurrence of events
Key: A C. Mean & median values of the given data
D. Relationship between two given variables
378. Which of the following variables is measured on the ordinal
scale? Key: D
A. Type of anemia
B. Severity of anemia 385. Graph showing relation between 2 variables is:
C. Hemoglobin level A. Scatter diagram
D. Serum ferritin level B. Frequency polygon
C. Picture chart
Key: B D. Histogram

379. Measurement of blood pressure is which type of data: Key: A


A. Nominal
B. Ordinal 16. Trends can be best represented by:
C. Interval A. Scatter diagram
D. Continous B. Bar diagram
C. Line diagram
Key: D D. Pie chart

380. Histogram is used to present which kind of the data: Key: C


A. Nominal
B. Continous 386. Histogram is used as method of group presentation for:
C. Discrete A. Qualitative data
D. All of the above B. Quantitative continuous data
C. Quantitative data- discrete type
Key: B D. Norminal data

381. All of the following methods can show relationship between Key: B
two variables except?
A. Histogram 387. The incidence of malaria in an area is 20, 20, 50, 56, 60,
B. Line diagram 5000, 678, 898, 345, 456. Which of these methods is the
C. Bar chart best to calculate the average incidence?
D. Scatter plot A. Arithmetic mean
B. Geometric mean
Key: C C. Median
D. Mode
382. A physician, after examining a group of patients of a
certain disease, classifies the condition of each one as Key: C
Normal’, ‘Mild’, ‘Moderate’ or ‘Severe’. Which one of the
following is the scale of measurement that is being adopted 388. In a bimodal series, if mean is 2 and median is 3, what is the
for classification of the disease condition? mode?
A. Normal A. 5
B. Interval B. 2.5
C. Ratio C. 4
D. Ordinal D. 3

Key: D Key: A

383. An investigator into the life expectancy of IV drug abusers 389. Central tendency is given by:
divides a sample of patients into HIV- positive and HIV- A. Mean
negative groups. What type of data does this division B. Median
constitute? C. Moe
A. Nominal D. All

400
397. If the birth weight of each of the 10 babies born in ahospital
Key: D in a day is found to be 2.8 kg, then the standarddeviation of
this sample will be:
390. Median is important for all except: A. 2.8
A. Blood pressure B. 0
B. Survival time C. 1
C. Incubation period D. 0.28
D. Health expenses
Key: B
Key: A
398. Among a 100 women with average Hb of 10 gm%, the
391. Most frequently occurring value in a group of data: standard deviation was 1, what is the standard error?
A. Mean A. 0.01
B. Mode B. 0.1
C. Median C. 1
D. Standard deviation D. 10

Key: B Key: B

392. Values are arranged in ascending and descending order to 399. The Hb level in healthy women has mean 13.5 g/dl and
calculate: standard deviation 1.5 g/dl, what is the Z score for awoman
A. Mean with Hb level 15.0 g/dl:
B. Mode A. 9.0
C. Median B. 10.0
D. S.D C. 2.0
D. 1.0
Key: C
Key: D
393. ‘Centile’ divides data into:
A. 100 equal parts 400. In a sample of 100 pregnant females, Mean haemoglobin
B. 4 equal parts level estimated was 10 gm% with a Standard deviation of 1
C. 10 equal parts gm%. What is the Standard error?
D. 20 equal parts A. 1 gm%
B. 10 gm%
Key: A C. 0.1 gm%
D. 100 gm%
394. 50th percentile is equivalent to:
A. Mean Key: C
B. Median
C. Mode 401. Following denotes measures of variability?
D. Range A. Range
B. Mean deviation
Key: B C. Standard deviation
D. Median
395. Standard deviation is the measure of:
A. Chance Key: A
B. Central tendency
C. Deviation from mean value 402. Most common deviation used in social medicine is:
D. None A. Mean
B. Range
Key: C C. Variance
D. Standard deviation
396. Standard deviation of means measures:
A. Non-sampling errors Key: D
B. Sampling errors
C. Random errors 403. There is a population of 20000 people with mean
D. Conceptual errors hemoglobin being 13.5 gm% having a normal distribution.
What proportion of population constitutes proportion more
Key: B than 13.5 gm%?
A. 0.25
B. 0.50
C. 1

401
D. 0.34 411. In the WHO recommended EPI Cluster sampling for
assessing primary immunization coverage, the age group of
Key: B children to be surveyed is:
A. 0-12 months
404. Square root of deviation is called: B. 6-12 months
A. Standard deviation C. 9-12 months
B. Standard error D. 12-23 months
C. Mean deviation
D. Range Key: D

Key: A 412. Sampling method used in assessing immunization status of


children under immunization program is:
405. Standard deviation does not depend on: A. Systematic sampling
A. Mean B. Stratified sampling
B. Median C. Group sampling
C. Range D. Cluster sampling
D. Sample size
Key: D
Key: B
413. In a study, people are separated into certain sub-groupsand
406. Standard error of mean is called as: then some are selected randomly from each of thesesub-
A. Standard deviation groups. What type of sampling is being done?
B. Mode A. Simple random sampling
C. Median B. Cluster random sampling
D. Variable C. Systematic random sampling
D. Stratified random sampling
Key: A
Key: D
407. The standard normal distribution:
A. Is skewed to the left 414. Simple random sampling is ideal for:
B. Has mean = 1.0 A. Vaccinated people
C. Has standard deviation = 0.0 B. Heterogenous population
D. Has variance = 1.0 C. Homogenous population
D. All of the above
Key: D
Key: C
408. A normal distribution curve depends on:
A. Mean and sample size 415. Stratified sampling is ideal for:
B. Range and sample size A. Heterogenous data
C. Mean and standard deviation B. Homogenous data
D. Mean and median C. Both
D. None
Key: C
Key: A
409. In normal curve:
A. Mean = 2 standard: deviation 416. In a study first schools are sampled, then sections,
B. Mean = Median andfinally students. This type of sampling is known as:
C. Mean = Variance A. Stratified sampling
D. Mean = 1 standard deviation B. Simple random sampling
C. Cluster sampling
Key: B D. Multistage sampling

410. Normal distribution curve: Key: D


A. Mean, median, mode are same
B. B/L symmetrical 417. Simple random sampling:
C. bell shape A. Provides least number of possible samples
D. All B. Equal chance to each for collection of certain
numberfor a sample
Key: D C. Picking every 5th or 10th at regular intervals
D. Sample represent, a corresponding strata of universe

Key: B

402
418. Appropriate statistical method to compare two meansis: 425. Chi-square test is for:
A. Chi-square test A. Standard error of mean
B. Student’s t-test B. Standard error of proportion
C. Odds Ratio C. Standard error of difference between 2 means
D. Correlation Coefficient D. Standard error of difference between proportions

Key: B Key: D

419. Appropriate statistical method to compare two proportions 426. Mean, Median and Mode are:
is: A. Measures of dispersion
A. Chi-square test B. Measures association between two variables
B. Student’s t-test C. Test of significance
C. Odds Ratio D. Measures of central tendency
D. Correlation Coefficient
Key: D
Key: A
427. The usefulness of a screening test depends upon its:
420. Chi-square test is used to measure the degree of: A. Sensitivity
A. Significance of difference between two proportions B. Specificity
B. Association between two variables C. Reliability
C. Correlation between two variables D. Predictive value
D. Agreement between two observations
Key: A
Key: A
428. When a diagnostic test is used in “series” mode, then:
421. True regarding Chi-square test is: A. Sensitivity increases but specificity decreases
A. Measure the significance of difference between two B. Specificity increases but sensitivity decreases
proportion C. Both sensitivity and specificity increase
B. Null hypotheses is equal D. Both sensitivity and specificity decrease
C. Does not test the significance
D. Tests correlation and regression Key: B

Key: A 429. What is the color-coding of bag in hospitals to dispose off


human anatomical wastes such as body parts?
422. Test of association between two qualitative variables isdone A. Yellow
by: B. Black
A. Chi-square test C. Red
B. Correlation D. Blue
C. Regression
D. None Key: A

Key: A 430. “Inertization” deals with:


A. Mixing biomedical waste with cement and other
423. About chi-square test, true is: substance before disposal
A. <0.001 is statistically significant B. incineration of biomedical waste with cement and other
B. Less no. of samples are associated with less error substance before disposal
C. Categories of data used in test need not be C. dumping of Biomedical waste in sanitary landfills
mutuallyexclusive and discrete D. screw feed technology to disinfect sharps
D. Tests correlation and regression
Key: A
Key: A
431. HIV (+) patient is being infused amphotericin B for fungal
424. A study measures a patient’s serum cholesterol before and systemic infection patient’s iv-cannula and tubing should be
after a new lipid-lowering therapy has been gives. What managed as which of the following:
type of significance test should be used to analyze the data? A. Disinfect in 1% hypo chlorite, put in blue bag for
A. Paired t-test destruction/shredding
B. Student’s t-test B. Put in Red bag for destruction/shredding
C. Chi-squared test C. Disinfect in 5% hypochlorite solution and put in
D. Pearson’s test yellow bag
D. Put in black bag for destruction/shredding
Key: A

403
Key: A
438. Yellow plastic bags containing biomedical wastes aretreated
432. What is the color-coding of bag in hospitals to dispose off by:
human anatomical wastes such as appendix? A. Autoclaving
A. Yellow B. incineration
B. Black C. Microwaving
C. Red D. shredding
D. Blue
Key: B
Key: A
439. Hospital waste product accounts:
433. What is the color-coding of bag in hospitals to dispose off A. Paper 40%& Glass 0%
waste sharps? B. Plastic 10%& Glass 4%
A. Yellow C. Infectious waste 30%
B. Black D. Rage 30%
C. Red
D. Blue Key: B

Key: D 440. Outdated cytotoxic drugs are best disposed by:


A. Disposa in municipal waste
434. Incineration is: B. Destruction and dumping in secured landfill
A. High temperature reduction process C. Store for months and burial
B. low temperature reduction process D. Autoclave
C. High temperature oxidation process
D. low temperature oxidation process Key: B

Key: C 441. Mercury is disposed by:


A. Controlled combustion
435. Which of the following biomedical wastes can be B. Deep burial
incinerated? C. Safely collect and re-use
A. Pressurized gas containers D. Chemical treatment
B. Radiographic wastes
C. PVC Key: C
D. Human anatomical wastes
442. All of the following statements regarding BiomedicalWaste
Key: D management are true except:
A. Human Anatomical waste is thrown in Yellow bag
436. Which of the following Biomedical wastes cannot be B. Blue bag waste is disposed by landfill
disposed off in Yellow Bags? C. Incineration ash is discarded in Black bag
A. Reactive chemical wastes D. Material in Red bag could be a source of contamination
B. Human anatomical wastes
C. Microbiology and Biotechnology wastes Key: B
D. dressings soiled with blood
443. A known HIV positive patient is admitted in an isolation
Key: A ward after an abdominal surgery following an accident. The
resident doctor who changed his dressing the next day found
437. A known HIV positive patient is admitted is admitted in an it to be soaked in blood. Which of the following would be
isolation ward after an abdominal surgery following an the right method of choice for discarding the dressing?
accident. The resident doctor who changed his dressing the A. Pour 1% hypochlorite on the dressing material and
next day found it to be soaked in blood. Which of the send it for incineration in an appropriate bag
following would be right method of choice of discarding the B. Pour 5% hypochlorite on the dressing material and
dressing? send it for incineration in an appropriate bag
A. Pour 1% hypochlorite on the dressing material and C. Put the dressing material directly in an appropriate bag
send it for incineration in an appropriate bag and send it for incineration
B. Pour 5% hypochlorite on the dressing material and D. Pour 2% lysol on the dressing material and send it for
send it for incineration in an appropriate bag incineration in an appropriate bag
C. Put the dressing material directly in an appropriate bag
and send for incineration Key: A
D. Pour 2% lysol on the dressing material and send it for
incineration in an appropriate bag 444. Incineration not done for:
A. Cytotoxic drugs
Key: A B. waste sharps

404
C. Human anatomical waste A. Anatomical waste
D. Cotton contaminated by blood B. sharps waste
C. Cytotoxic drugs
Key: B D. Radioactive waste

445. Discarded expired medicines are thrown into: Key: B


A. Blue bag
B. Black bag 453. True about Inertization all except:
C. Yellow bag A. Mixing biomedical waste with cement
D. Red bag B. used for pharmaceutical waste
C. Contaminates water sources
Key: B D. Not useful for infectious waste

446. Incineration is done for: Key: C


A. Waste sharps
B. Human anatomical waste 454. Disposal mechanism for Black colour coded biomedical
C. Radiographic waste waste bag is
D. Used batteries A. incineration
B. dumping
Key: B C. shredding
D. landfill
447. Amount of infectious waste among hospital waste is:
A. 1.5% Key: D
B. 4.5%
C. 25% 455. Post disaster (earthquake) in Pakistan, which of
D. 12% thefollowing vaccines is recommended by WHO?
A. typhoid
Key: A B. Cholera
C. tetanus
448. Disposal of placenta at primary health care is: D. None of the above
A. Dry burning
B. Deep burial Key: D
C. Boiling
D. Treat with bleaching powder and burial 456. During a disaster, rapidly classifying the injured on the
basis of likelihood of their survival with prompt medical
Key: D intervention, is a part of:
A. Search, rescue and first aid
449. Plastic cover of syringes are disposed in: B. triage
A. Red bag C. tagging
B. Yellow bag D. disaster mitigation
C. Black bag
D. Blue bag Key: B

Key: C 457. Most commonly reported disease in the post disasterperiod


is:
450. Waste sharps should be disposed in: A. Acute Respiratory infections
A. Black bag B. Gastroenteritis
B. Yellow bag C. tetanus
C. White bag D. Malaria
D. Yellow bag
Key: B
Key: C
458. As per the most common classification of Triage system
451. True about Incinerator is/are: that is internationally accepted, the colour code that
A. Red bag can be incinerated indicates high priority treatment or transfer is:
B. No pre-treatment required A. Black
C. Yellow bag must be incinerated B. Yellow
D. sharps must not be incinerated C. Red
D. Blue
Key: C
Key: C
452. Incineration is not done for:

405
459. Which colour-coded person is given first preference
indisaster triage? Key: C
A. Red
B. Black 467. During massive disaster what should be done first?
C. Yellow A. Search and rescue, first aid
D. Green B. triage
C. stabilization of victims
Key: A D. Hospital treatment and redistribution of patients to
hospital if necessary
460. Black color in triage is:
A. death Key: A
B. transfer
C. High priority 468. In a disaster management triage, patients who need surgery
D. low priority within 24 hours, are categorized under which color category:
A. Red
Key: A B. Green
C. Blue
461. All vaccines are NOT given in disaster, except: D. Black
A. Cholera
B. tetanus Key: C
C. Measles
D. tetanus 469. Lead poisoning in industries commonly occurs by:
A. inhalation
Key: C B. ingestion
C. skin absorption
462. True about triage is: D. Conjunctival route
A. Yellow-least priority
B. Red-morbidity Key: A
C. Green-ambulatory
D. Blue-ambulatory 470. Inhalation of sugarcane dust could cause:
A. Bagassosis
Key: C B. Byssinosis
C. tobacosis
463. Which is the calamity with most amount of damage? D. farmer’s lung
A. flood
B. earthquake Key: A
C. landslides
D. Volcanoes 471. Maximum permissible level of whole body occupational
exposure to ionizing radiation is:
Key: A A. 1 rem per year
B. 3 rem per year
464. Natural disaster causing maximum deaths: C. 5 rem per year
A. Hydrological D. 15 rem per year
B. Meterological
C. Geological Key: C
D. fires
472. Respirable dust, responsible for pneumoconiosis, has asize
Key: A limit of:
A. < 1 micron
465. Triage is: B. < 5 micron
A. A concept in trauma C. < 10 micron
B. A method of breast lump diagnosis D. < 100 micron
C. An investigation for duodenum and pancreas
D. Management of old age health problems Key: B

Key: A 473. Which of the following are associated with Bladder cancer:
A. Nickel
466. Epidemics after disaster are caused by all except: B. Naphthylamines
A. leptospirosis C. Arsenic
B. Rickettsiosis D. lead
C. leishmaniasis
D. Acute respiratory infection Key: B

406
B. Heat cramps
474. All of the following features are suggestive of asbestosis C. Heat exhaustion
except: D. Heat hyperpyrexia
A. Occurs within five years of exposure
B. the disease progresses even after removal of contact Key: B
C. Can lead to pleural mesothelioma
D. sputum contains asbestos bodies 482. Benzene associated with cancer of the
A. Skin
Key: A B. Lung
C. Bladder
475. Bagassosis can be prevented by spraying: D. Leukemia
A. 10% acetic acid
B. 5% acetic acid Key: D
C. 1% propionic acid
D. 2% propionic acid 483. Wrist drop may be caused as industrial hazard in:
A. Battery industry
Key: D B. Gas industry
C. Asbestos, industry
476. Bagassosis is a pneumoconiosis caused by inhalation of: D. Aniline industry
A. sugarcane dust
B. Cotton Key: A
C. Coal particles
D. Asbestos 484. “Snow-storm” appearance are seen in:
A. Anthracosis
Key: A B. silicosis
C. Byssinosis
477. Following are the chemical agents, which causes D. Bagassosis
occupational dermatitis by local irritation except:
A. Rubber Key: B
B. X-rays
C. lime 485. Exposure to cotton dust causes:
D. ether A. Byssinosis
B. Bagassosis
Key: D C. silicosis
D. Asbestosi
478. Most common mode of lead poisoning is:
A. ingestion Key: A
B. dermally
C. inhalation 486. ‘Bagassosis; is an occupational disease of the lung caused
D. faecal-oral by inhalation of:
A. Cotton
Key: C B. Jute
C. sugarcane dust
479. Which occupational exposure may cause sterility infemales: D. Coal
A. Lead & Agricultural insecticides
B. Carbon monoxide Key: C
C. Mercury& silica
D. None 487. Monday fever is associated with:
A. Bagassosis
Key: A & D B. Byssinosis
C. Asbestosis
480. Silicosis occurs with: D. silicosis
A. Cotton
B. Sugarcane fibres Key: B
C. Carbon particles
D. Silica fibres 488. Silicosis affects:
A. lower lobes of lungs
Key: D B. Both
C. Hilum of lungs
481. A person working in hot environment who consumes more D. upper lobes of lungs
H2O without salt is likely to develop:
A. Heat stroke Key: D

407
B. 4 weeks
489. Respirable dust for pneumoconiosis: C. 6 weeks
A. 0.1–5 m D. 8 weeks
B. 5–10 m
C. 10–15 m Key: A
D. 15–20 m
497. Best way to dispose hospital waste off
Key: A A. Burning
B. Incineration
490. Dust particle, in a industry is a: C. In a landfill
A. Biological hazards D. Recycling
B. Chemical hazards
C. Physical hazards Key: B
D. Mechanical hazards
498. In which stage of the demographic cycle is Pakistan
Key: currently?
A. High stationary
491. Bagassosis is due to long term inhalation of: B. Late expanding
A. Cotton fibres C. Early stationary
B. sugar cane dust D. Low stationary
C. thermophilic bacteria
D. silica Key: B

Key: B 499. In a demographic cycle Pakistan belongs to Late/still


expanding corresponds to which stage?
492. Which of the following is occupational lung disease but not A. First stage
pneumoconiosis? B. Second stage
A. Brucellosis C. Third stage
B. silicosis D. Fourth stage
C. Anthracosis
D. Byssinosis Key: C

Key: A 500. Late expanding stage of population in Pakistan is due to?


A. Birth rate stationary death rate continues to fall
493. Bagassosis is caused by: B. Death rate declines faster than birth rate
A. Cotton dust C. Birth rate declines, death rate same
B. sugar cane D. Birth rate is less than birth rate
C. Coal dust
D. silica Key: B
Key: B 501. The fourth stage of the demographic cycle is:
A. Declining
494. Occupational exposer to cotton dust causes: B. Early expanding
A. Byssinosis C. High stationary
B. Bagassosis D. Low stationary
C. Anthracosis
D. Pneumoconiosis Key: D
Key: A 502. 3rd stage of demography indicates:
A. High birth rate and high death rate
495. Amniocentesis to detect chromosomal abnormalities can be B. Death rate begins to decline
done as early as: C. While the birth rate remains unchanged
A. 14th week of gestation D. Birth rate tends to fall and death rate declines still
B. 18th week of gestation further
C. 22nd week of gestation
D. 26th week of gestation Key: D
Key: A 503. Early expanding stage is denoted by:
A. Decreased birth rate and decreased death rate
496. Under passive surveillance for tuberculosis, indication for B. Increased birth rate and Increased death rate
sputum microscopy cough is for weeks or more: C. Decreased birth rate and Increased death rate
A. 2 weeks D. Unchanged birth rate and decreased death rate

408
A. Bleeding
Key: D B. Pain
C. Pelvic infection
504. Pakistan is in which stage of population growth: D. Ectopic pregnancy
A. Late expanding stage
B. Early expanding stage Key: B
C. High stationary stage
D. Low stationary stage 512. Characteristics of an ideal candidate for copper –T insertion
include all of the following except:
Key: A A. Has borne at least one child
B. Is willing to check IUD tail
505. Age pyramid of Pakistan is: C. Has a history of ectopic pregnancy
A. Broad at base and narrow at apex D. Has normal menstrual periods
B. Broad from base to apex
C. Broad at apex and narrow at base Key: C
D. All
513. Most common side effect leading to IUD removal is:
Key: A A. Bleeding
B. Pain
506. Barrier methods are all except: C. Infection
A. Diaphragm D. Uterine perforation
B. Foam tablets
C. Vaginal-sponge Key: B
D. Lippes loop
514. Copper – T is preferably inserted postnatal, after:
Key: D A. 2 weeks
B. 4 weeks
507. Natural Family Planning does not include: C. 5 weeks
A. Terminal methods D. 8 weeks
B. Basal Body Temperature Method
C. Cervical Mucus Method Key: D
D. Symptothermic Method
515. Which of the following IUDs do not require to bechanged
Key: A every 3-5 years?
A. CuT 220 B
508. Which of the following Natural method of contraception is B. CuT ML-375
most effective? C. CuT 380 A
A. Calendar method D. CuT ML-250
B. Billing method
C. Symptothermic method Key: C
D. Basal body temperature method
516. Crude birth rate is a simplest measure of fertility because it
Key: C includes:
A. Total population
509. the most common side effect of IUD insertion is: B. Mid-year population
A. Bleeding C. Live births only
B. Pain D. Pre-term births
C. Pelvic infection
D. Ectopic pregnancy Key: B

Key: A 517. Birth rate is:


A. Live birth/1000 mid-year population
510. All are true about Progestasert except: B. Birth/1000 mid-year population
A. Progestasert releases 65 mcg progesterone per day C. Live birth/10000 mid-year population
B. Progestasert contains 38 mg progesterone D. Live birth/1000 population
C. Progestasert is implanted subdermally
D. Progestasert is a T-shaped device
Key: A
Key: C
518. 3rd generation IUCD acts by:
511. The most common side effect of IUD insertion, which A. Strong anti-fertility effect of metallic copper
requires its removal is: B. By altering the composition of cervical mucous

409
C. Hormonal effect on mucosa of endometrium Key: D
D. Enhanced cellular response on endometrium
526. Which of the following is/ are benefits of combined OCPs
Key: C use?
A. Ovarian cysts
519. Absolute contraindication of IUCD is: B. Fibrocystic disease of breast
A. Anemia C. Ectopic pregnancy
B. Diabetes D. All
C. PID
D. Hemorrhage Key: D

Key: C 527. Serious complication of oral contraceptive is:


A. Leg vein thrombosis
520. Multi load device refers to: B. Headache
A. First generation IUCD C. Break through bleeding
B. Second generation IUCD D. Breast tenderness
C. Oral contraceptive pills
D. Barrier contraceptives Key: A

Key:B 528. All are non-contraceptive advantages of oral contraceptive


pills except:
521. Besides pregnancy the oral contraceptive protect against all A. Pelvic inflammatory disease
except: B. Hepatic adenoma
A. Fibroadenoma breast C. Benign breast cancer
B. Iron deficiency anemia D. Anemia
C. Ovarian cancer
D. Hepatocellular adenoma Key: B

Key: D 529. The side effects like irregular bleeding, depression


areassociated with use of
522. Which one of the following is NOT an A. Mini pill
absolutecontraindication for oral contraceptive pills? B. OC pills
A. Nursing mothers C. Mifepristone
B. Cancer of breasts D. None
C. Cardiac abnormalities
D. History of thrombo-embolism Key: A

Key: A 530. The absolute contraindication for prescribing


normalcontraceptive pills in a woman of reproductive
523. Contraindication of O.C.P agegroup is
A. Liver disease A. Epilepsy
B. Renal disease B. Diabetes mellitus
C. Epilepsy (A, C, D) C. Milk hypertension
D. All D. Congenital hyperlipidemia

Key: D Key: D

524. Contraceptive to be avoided in epilepsy is: 531. All of the following can be used as emergency contraceptive
A. Combined OCPs measures except:
B. Condoms A. Female condoms
C. IUDs B. IUD
D. Post-coital (emergency) pills C. Minipill
D. Yuzpe and Lancee
Key: A
Key: A
525. Non-contraceptive benefits of combined oral pills is/ are
reduction of: 532. Yuzpe and Lancee Method is used for:
A. Iron deficiency anaemia A. Sterilization with ‘No Scalpel Technique’
B. Ovarian cancer/Ovarian cysts B. Emergency contraception with OCPs
C. PID C. Emergency contraception with IUDs
D. All D. Evaluation of newer contraceptives

410
Key: B B. OCP’s
C. IUCD
533. Which of the following is not used as an D. Inj. DMPA
emergencycontraceptive?
A. LNG- Intrauterine device Key: A
B. Oral LNG
C. CuT-Intrauterine device 541. Ideal contraceptive for a newly married couple is:
D. Oral Mifepristone A. OCP
B. Barrier method
Key: A C. IUCD
D. Natural methods
534. Post coital contraceptives are all except:
A. Norgestrel Key: A
B. OCPs
C. RU-486 542. Most cost effective contraceptive is:
D. Copper-T A. Vasectomy
B. Tubectomy
Key: A C. Cu-T
D. OCP
535. Increased incidence of ectopic is associated with allexcept:
A. IUD Key: A
B. Combined oral pills
C. Menstrual regulation
D. Safe period method

Key: B

536. Most cost effective family planning method is:


A. Vasectomy
B. Tubectomy
C. Copper T
D. Oral pills

Key: A

537. Conventional Contraceptives are those which:


A. Were discovered before 1960
B. Require action after intercourse
C. Require action at time of intercourse
D. Require action before intercourse

Key: C

538. Best contraceptive for a newly married healthy couple:


A. Barrier method
B. IUCD
C. Oral contraceptive pills
D. Natural methods

Key: C

539. Ideal Contraceptive for lactating women:


A. POP
B. IUCD
C. Lactation amenorrhoea
D. Barrier methods

Key: B

540. Ideal contraceptive for a couple who are living separately in


two cities and meets only occasionally:
A. Barrier methods

411

Вам также может понравиться